.
. centre de ressources dilemmes et doutes le visage humain de mathématiques Qui sommes-nous Problème de mois activités de promotion babillard
Centrale des maths - centraledesmaths.uregina.ca
Dilemmes & doutes
« D & D »
. .
topic card  

Sujet:

ap

liste de
sujets
. .
nouvelle recherche

483 articles trouvés pour ce sujet.
 
Page
1/1
Combien de bonds aura-t-il besoin 2012-04-10
zachary pose la question :
Un lapin fait toujours un bond de 3m suivi d'un bond de 5m. S'il maintient ce rythme de combien de bonds aura-t-il besoin pour franchir une distance de 584m ??
Claude Tardif lui répond.
la capcite en metre cube de ma remorque 2008-03-22
pages pose la question :
je voudrai connaitre la capcite en metre cube de ma remorque les dimensions font 124-95 -34 merci
Pierre-Louis Gagnon lui répond.
Problème avec rapport point, prix... 2005-09-08
Nico pose la question :

Le prix d'un diamant est proportionnel au carré de son poinds. Un diamant de 0.45 g vaut 7500euros.

1. Combie coute un diamant de 0.693 g ?

2. Quel est le poids d'une diamant valant 45000 euros ?

Je vous demande pas de le raisoudre à ma place mais j'aimeré comprendre comment je peut calculé le rapport entre le point et le prix.


Claude Tardif lui répond.
La traduction anglais 2002-05-27
Une etudiant pose la question :
je voudrais savoir la traduction anglais de :

homothétie (mot de niveau intermédiaire (6-9))
apothème (mot de niveau internédiaire (6-9))

parce que je ne les trouve pas.

Chris Fisher at Claude Tardif lui répond.
l'équation d'une rotation dans un graphique cartésien 2001-11-14
Ghaith pose la question :
je d6sir que vous me rafraichissez un peu la mémoire s.v.p. j'aimerai savoir l'équation d'une rotation dans un graphique cartésien merci
Claude Tardif lui répond.
la pantographie 1999-11-18
Erick et Christine pose la question :
Nous recherchons la définition de pantographie voltaïque, que représente cette "figure" ? est-ce qu'on peut en définir une unité de mesure ?
Claude Tardif lui répond.
logarithme neperien 1999-11-10
Louise Kieffer pose la question :
D'où provient la valeur e ( 2,7....) des logarithmes népériens ?
Claude Tardif lui répond.
Application surjective 1999-09-30
Charass pose la question :
je suis étudiant en premiere année de deug et je cherche apres une definition si vous pouviez m'aider, ça serait sympa. qu'est ce qu une application subjective?
Claude Tardif lui répond.
A graph with 100 vertices and one edge 2022-01-11
Maftuna pose la question :
A graph has 100 vertices and only one edge. How many connected components does it have?
Penny Nom lui répond.
More on a roll of paper 2021-05-07
Stephen pose la question :
Can you help with the equations to calculate the length of paper required to achieve a target outside diameter when wrapped around a core please?

The inside diameter of the cardboard core is 76mm
The thickness of the wall of the cardboard core is 5mm
The thickness of the paper is 138microns
The desired outside diameter of the finished roll is 320mm

I hope you can help me with a solution, Thank you

Harley Weston lui répond.
Are all quadrilaterals the same? 2020-05-27
eva pose la question :
are all quadrilaterals the same?
Penny Nom lui répond.
The volume under an an uneven surface 2020-05-01
Solon pose la question :
I am trying to fill an area of floor which has an uneven surface. I need to know the volume of fill. If you imagine a rectangular room (4 corners) that measures 189 inches by 240 inches and the depth at each corner is different.
Let us say corner #1 has a depth of 1", corner #2 has a depth of 0", corner #3 has a depth of 2" and corner #4 has a depth of 4".
How would i find the volume of said floor area?

Any assistance would be greatly appreciated!

Harley Weston lui répond.
A straight line graph 2020-02-26
Penehafo pose la question :
I have a problem of drawing a straight line graph by the equation y=2x-1
Penny Nom lui répond.
A system of inequalities 2020-02-01
Isaac pose la question :
Austin has x nickels and y dimes, having a maximum of 15 coins worth a minimum of $1 combined. No more than 4 of the coins are nickels and no less than 11 of the coins are dimes. Solve this system of inequalities graphically and determine one possible solution.
Penny Nom lui répond.
Investigating y = (-2)^x 2020-01-13
Gonzalo pose la question :
This is not precisely a maths question, but it is formulated based on my maths curiosity. Fidgetting with my new graphic calculator, I started graphing things and had the idea to graph $y=(-2)^x.$ The result surprised me, and I thought a little bit about it, stored it on the back of my brain, and promised myself to look deeper into it someday.
Harley Weston lui répond.
Two people are clapping their hands 2019-12-06
Yukti pose la question :
Two people are clapping their hands, but with different frequencies. Ramya claps her hands 17 times per 3 minutes, and Kumar claps his hands 31 times per 5 minutes. They start clapping at the same time. How many times does each one clap before they clap together at the same time again?
Penny Nom lui répond.
The area of a shape 2019-08-14
Lymp pose la question :
Hi, for math we are calculating the area of shapes and my teacher wanted me to research how to calculate the area of shapes so i was wondering if you could tell me how t calculate the area of any shapes. Please tell me how to calculate a cross, parallelogram and a polygon
Penny Nom lui répond.
Volume of a spherical cap 2019-07-12
Shanto pose la question :
In a jar of water, we kept a sphere with radius r. Then 75% of its volume went under water. Find out the distance of the top of the sphere from the surface of the water.
Penny Nom lui répond.
The volume of a wall 2019-05-08
Peter pose la question :
I’m building a dry stack stone wall which is 36” wide at the base, 42” tall and 24” wide at the top. What would be the volume for a 36” long segment ?
Thanks, Peter

Penny Nom lui répond.
When is John's next birthday on a Friday? 2019-03-15
Darek pose la question :
The question is the following:
Last Friday 4th of April of 2014 John was 40 years old. How old will John be the next time when his birthday fell on Friday?
Thanks for your help

Penny Nom lui répond.
A circle with a circular hole 2019-03-11
Sue pose la question :
My little one is wondering if a circle with a circular hole could be described as an irregular semicircle.
As it has 2 sides but is not the standard shape. Could you share your thoughts ??

Penny Nom lui répond.
y = |x| - 1 2018-11-30
alexis pose la question :
what does this mean y=|x|-1
Penny Nom lui répond.
The area of a trapezoid 2018-09-24
An other pose la question :
AB-272 feet: AC 690 feet; CD-330 Feet; DA-669Feet Calculate the area in feet and Acres.
Penny Nom lui répond.
Robbie and fran run laps 2018-08-26
khansa pose la question :
robbie runs 3 laps for unknown number of days. fran runs 3 times as many laps each day for unknown number of day. what is the formula for their combined total number of laps
Penny Nom lui répond.
The volume of a berm 2018-06-04
Mike pose la question :
Have a berm that is 700’ long, 29’ y’all and needs a 20’ top with 5-1 slope on one side and 3-1 slope in the other
Penny Nom lui répond.
Converting inches to centimeters 2018-05-25
Jason pose la question :
Hi!
Is there a method to approximately convert inches to cm in my head without using a calculator?

Thanks! Jason

Penny Nom lui répond.
The angles of a trapezoid 2018-04-14
Gautham pose la question :
In trapezoid ABCD, sides AB and CD are parallel, \angle A = 2angle D, and angle C = 3angle B. Find angle A.
Penny Nom lui répond.
Two regular pentagons 2018-03-19
Kip pose la question :
The ratio of the corresponding apothems of two similar regular pentagons is 5:7 What is the ratio of their areas?
Penny Nom lui répond.
Solve sinX=0.703X for X 2018-03-13
PARAM pose la question :
sinX=0.703X
Penny Nom lui répond.
Graphing y = 2x - 1 2018-03-12
Himanshu pose la question :
Hi I Himanshu I having 1 doubt on 1 question this question of graph how to draw y=2x-1
Penny Nom lui répond.
The graph of graph y= -2x-1 2017-11-11
Natividad pose la question :
How do you graph y= -2x-1?
Penny Nom lui répond.
The distance between the origin and a moving point on a graph 2017-10-16
Paulina pose la question :
Find the rate of change of the distance between the origin and a moving point on the graph of y=x^2 +1 if dx/dt=2 centimeters per second
Penny Nom lui répond.
A trapezoid inscribed in a circle 2017-05-17
Kameron pose la question :
I have been given a challenge problem that states that Diameter AB is drawn in a circle if 10 inch. Chords AC and BD are drawn so that each is of length 12inch and ACDB is a trapezoid. Find the height, in inches, of the trapezoid
Penny Nom lui répond.
The height of a isosceles trapezoid 2017-04-03
Riham pose la question :
Hi
How can I find the height in an isosceles trapezoid if I have the measurements of all of its sides ? Thank u in advance.

Penny Nom lui répond.
Covering the ceiling of a garage 2017-02-20
DeAnna pose la question :
I am trying to drape a 15 x 30 garage with yards of fabric. If the yard of fabric is 60 inches wide, would 10 1/4 yards be enough for my space?
Penny Nom lui répond.
A pie chart 2016-12-05
vickie pose la question :
Determine the central angle needed to form a pie chart for the following housing characteristic data: 63% owner occupied, 27% renter occupied, and 10% vacant
Penny Nom lui répond.
2^x= -2x - 11 2016-11-27
Kathy pose la question :
I don't know how to start this problem:
2^x= -2x - 11

Penny Nom lui répond.
The amount of material remaining on a roll 2016-11-11
Yoh pose la question :

Question from Yoh:

Hello,
I am trying to find impressions on a roll (either full roll or partial).
Let's say I have the following information.
- Outer Diameter of roll
- Inner Diameter of roll (cardboard core)
- thickness per layer
- Each cut off length (impression length)

Now let's say a roll has a 40in outer diameter, the cardboard core has a 3.75in diameter and a thickness of .002. Each impression has a cut off of 14inches.
With this the roll will have approximately 2,700 impressions. How would I find the remaining impressions if the outer diameter of the roll is only 6.5inches?

Thank you.


Penny Nom lui répond.
A stained glass cone lamp 2016-04-09
Edwin pose la question :
In making a 16" dia. cone lamp (stained glass), how many square feet of glass do I need.
Penny Nom lui répond.
A block wall 2016-03-15
Robert pose la question :
I have a wall 40 ft. long and 4 ft. tall as well as another one that is 30 ft by 3.5 ft. tall that I am trying to build. how many 6 inch by 12 inch landscaping blocks do I need?
Penny Nom lui répond.
The diameter of the top of a truncated cone 2016-01-24
Peter pose la question :
I am trying to calculate the diameter of a truncated cone given one diameter the height of the cone and a 10% taper from one end to the other. For example a butter churn is 18" tall and 9" in diameter at the base. the sides need to slope inward at 10%
What is the diameter at 9" and 18"

Penny Nom lui répond.
The volume of a silo 2016-01-18
Clinton pose la question :
Can you show me how to calculate the ground corn capacity in a steel silos of 10' x 10' high and 10' x 20' high and 10' x 24' high and 10' x 28' high
Penny Nom lui répond.
A triangular shaped garden 2016-01-13
Demetrie pose la question :
Dwayne's garden is triangle-shaped with two equal sides and a third side that is 4 ft more than the length of an equal side. If the perimeter is 49 ft, how long is the longest side?
Penny Nom lui répond.
The perimeter of a triangular plot of land 2016-01-11
Janelle pose la question :
I need to fence off 6.5 acres. The plot is triangular shape. How many feet would I be fencing off?
Penny Nom lui répond.
The height of a parabolic arc 2015-12-30
Tom pose la question :
Is there an algebraic means to determine the highest point of a parabolic arc if the base and perimeter are known?
Penny Nom lui répond.
The angles of a triangle 2015-12-17
Faith pose la question :
Does the measure of angle determine the length of its side? For example two angles are congruent then the sides are also congruent because from my understanding the angle determine the shape of triangle.
Penny Nom lui répond.
The positive root of sin(x) = x^2 2015-12-13
Kemboi pose la question :
Find the positive root of the equation sin(x) = x^2
Penny Nom lui répond.
The area of a domed roof 2015-11-13
Brandon pose la question :
I have a tank with a 13' radius that has a domed roof of 3.5' tall How do I figure out the area in SQFT?
Penny Nom lui répond.
Why are bags of soil measured in litres? 2015-06-19
Cheryl pose la question :
Why are bags of soil measured in litres?
Penny Nom lui répond.
A piecewise function 2015-04-25
uday pose la question :

f(x)={x^2+3x+2 / x+2, x(not equal to)=-1
{4, x=-1
how to find domain and how the graph looks like


Penny Nom lui répond.
Two cars approaching an intersection 2015-04-16
Engabu pose la question :
Car A is traveling west at 50km/h & car B is traveling north at 60km/h. both are headed for the intersection of the two roads. At what rate are the cars approaching each other when car A is 3km & car B is 4km from the intersection?
Penny Nom lui répond.
A circle graph 2015-04-06
Davis pose la question :
i don't know how to answer this question. please help!

a survey about student government shows the following results

110 like the program
120 think the program is unnecessary
210 plan on running for student government next year

if this information is put in a circle graph,what is the central angle for the students who like the program?

Penny Nom lui répond.
The volume of a trench 2015-03-11
Kim pose la question :
A trench 60m long and 15m deep, 12m wide at the top and 8m wide at the bottom.How many cubic meters of earth have been removed?
Penny Nom lui répond.
Is a square a trapezoid? 2015-01-26
Katie pose la question :
Can a trapezoid sometimes be a square?
Penny Nom lui répond.
The length of a roll of paper 2015-01-12
Peggy pose la question :
roll of paper 3ft wide and roll equals 500 sq ft how long would the roll be?
Penny Nom lui répond.
A truncated, square-based pyramid 2014-11-23
Hannah pose la question :
A designer vase has the shape of a truncated, square-based pyramid. The base of the vase is a square with a side length of 15 cm. The area of the square opening is 70.56 cm2. Each of the four sides is a trapezium with slant sides 9 cm long. Find (to the nearest square centimetre) the total surface area of the vase.
Penny Nom lui répond.
An arithmetic progression 2014-11-19
Gbenga pose la question :
In an A.P the difference between 8th and 4th term is 20. The 8th term is 1\2 times the 4th term . Find arithmetic progression..
Penny Nom lui répond.
A piecewise graph 2014-05-10
Zoe pose la question :
Lightning Energy charges residential users for each unit of electrical energy bought from them each quarter, according to the scale below:
0-500 units cost 17 cents per unit
500-1500 units cost $85 plus 10 cents for each unit in the excess of 500
1500 or more units cost $185 plus 9 cents for each unit in excess of 1500
Sketch a graph showing the charge C as a function of U for up to 2000 electricity units

Penny Nom lui répond.
The surface area of a circular dome 2014-04-10
Shafiqah pose la question :
Is this a dome's surface area formula??
{{2 × π r × h square units}}
Is the surface area of the floor for the dome is calculate too in this formula?

Thanks for answering. =)

Robert Dawson and Penny Nom lui répond.
Graphing piecewise functions 2014-03-29
Rayven pose la question :
Hi! I'm in eighth grade, taking ninth grade algebra 1. I'm confused as to how to graph piecewise functions. I know that you have already answered a question similar to this (I did my research first) but it didn't completely help me on my homework. I have to graph piecewise functions for the specified domains, and create a table for the absolute values. I know that two bars around a number means absolute value (two bars around -2 makes it +2) , but how do I graph and chart the absolute value for the following:

f(x)= |x+3 | for -5≤x≤3

And then graph and chart: (on a separate graph):

f(x)= {x if x≤0
{x+1 if x <0

thank you!
~Rayven

Penny Nom lui répond.
Two overlapping arcs in a square 2014-03-15
Jean pose la question :
I have a square with side 4 cm. There are two overlapping arcs going from vertex to diagonal vertex. The other two vertices are the center of the arcs, which are shaded. How do I find the area of the shaded arcs? The overlapping arcs when shaded resemble a long thin football
Thank you for your help.

Penny Nom and Walter Whiteley lui répond.
A circle graph 2014-03-06
Caitlyn pose la question :
I need to find the measure of the central angle that represents the amount of time spent on each activity.
My question includes a circle graph with the following: Sleep 31%, Other 15%, Entertainment 18%, Errands 7%, Work 20%, and Food 9%.

Penny Nom lui répond.
The graph of y=2x+4 2014-02-18
tyler pose la question :
y=2x+4 graph
Penny Nom lui répond.
The graph of f(x)=x^2-4 2014-02-07
Allison pose la question :
Graph f(x)=x^2-4 and give the five point of the x/y intercepts.
Penny Nom lui répond.
A rectangular shaped property 2014-01-17
Donna pose la question :
If I I have property that is rectangle shaped. I I know the ends are 100 feet how many feet would the sides be to make 5.4 acres
Penny Nom lui répond.
On what day of the week was July 4, 1904? 2014-01-11
Madi pose la question :
Hi guys,
I have a question. July 4, 1903, was a Tuesday. On what day of the week was July 4, 1904? If you could give me an equation that would work for any question like this, that would be GREAT!
Your fellow math scholar (in training),
Madi

Penny Nom lui répond.
What does y= f(x) actually mean? 2013-12-31
John pose la question :

I don't understand how to pick coordinates for y=f(x).

I took a look at your answer to a previous question here:

http://mathcentral.uregina.ca/QQ/database/QQ.09.00/monica2.html

What does y= f(x) actually mean?


Penny Nom lui répond.
A circle insubscribed in an isosceles trapezoid 2013-12-08
Bob pose la question :
A circle is insubscribed in an isosceles trapezoid, with parallel lengths of 8cm and 18cm. What is the lengths of sloping edges and why?
Robert Dawson lui répond.
How do I graph y=-2x-1? 2013-12-07
cindy pose la question :
How do I graph y=-2x-1?
Penny Nom lui répond.
A sloped concrete wall 2013-10-30
Brad pose la question :
We are planning on pouring concrete in a wall form that is sloped. One end is 73 inches tall, and it tapers to 24 inches tall over a distance of 348 inches. It is flat on bottom and sides and is 12 inches thick. Thanks.
Penny Nom lui répond.
2^x . 5^3 = x^4 2013-09-05
Zuhdina pose la question :
Hi, I got a question, 2^x . 5^3 = x^4 I want to know the value of x without using calculator, I've already done some questions in logarithm and I always stuck in the same model equation-I think if someone shows me the way, I can do the other 'stuck equations'
Penny Nom lui répond.
Two overlapping circles 2013-05-22
Alexandra pose la question :
There are two overlapping circles. The two non-overlapping regions have areas A and B. As the area of overlap changes, the values of A and B also change. Prove that no matter how big and small the overlap is, the difference between A and B is always the same.
Penny Nom lui répond.
How can we draw the equation "x+y=1" in 3d? 2013-05-03
shimaa pose la question :
how can we draw the equation "x+y=1" in 3d
Robert Dawson lui répond.
Solve sin 3x = -0.1254 with x between o and 360 degrees 2013-02-21
David pose la question :
sin 3x = -0.1254 0
Harley Weston lui répond.
Solve sin 3x = -0.1254 with 0 2013-02-21
David pose la question :
sin 3x = -0.1254 0
Harley Weston lui répond.
An arithmetic progression 2013-02-06
loberto pose la question :
the 3rd term of an a.p is 10more than the 1st term,while the 5th term is 15more than the 2nd term,find the sum of the 8th and 15th terms of the a.p if the 7th term is 7times the 1st term
Penny Nom lui répond.
Plotting y=f(x)+2 2013-01-21
Gavin pose la question :
how would you plot y=f(x)+2
Penny Nom lui répond.
A bipartite graph 2012-10-29
A student pose la question :
Suppose that G be a bipartite graph with maximum degree of k.

Prove that:

1)Exists a K-regular bipartite graph that G be subgraph it(H)

Claude Tardif lui répond.
A label to cover a plastic cup 2012-10-23
Kevin pose la question :
I'm trying to make a label to cover the entire outer area or a plastic cup. I know there must be a way to figure out the dimensions needed, but I can't seem to figure it out. The circumference of the bottom of the cup is 21.4cm and the circumference at the top of the cup is 29.8cm. The cup is 14.5cm tall. What should the height of the arc from the plane connecting the two ends of the 21.4cm arc. I attached a diagram where x is the value I'm looking for. I'm guessing there is some simple relationship between the length of a line and the arc needed to turn that line into a perfect circle, but I don't know what it is. Can you figure this out and share it with me? Thanks.

-Kevin

Penny Nom lui répond.
A proof by contraposition 2012-10-19
Rahul pose la question :
I am not able to understand the following,
To prove that if for all e>0, |x|0, then |x|>=e. I understand the approach very well but I do not understand why if |x|=e then |x|>=e. If it is so then why not |x|= Thanks in advance!'
Rahul.

Penny Nom lui répond.
The number of solutions of sinx = x^2 + x +1 2012-09-01
anamika pose la question :
number of solutions of

sinx = x^2 + x +1
graphically?

Penny Nom lui répond.
Making a wind sock 2012-08-28
John pose la question :
I am trying to build a wind sock and need to be able to lay the shape out on cloth. I need the wind sock front opening (diameter) to be 3 1/2" and the rear opening diameter to be 1". The windsock needs to be 9 1/2" long. I tried using the example of the person trying to make a crayfish trap but got confused and could not figure out my numbers. Any help would be greatly appreciated.

Thanks

John

Penny Nom lui répond.
A tapestry rod on a curved wall 2012-08-14
Marlyn pose la question :
I have a curved wall with a radius of 6'. I am trying to have a 36" rod made to hang a tapestry and need to figure out the degree measure of the arc. Can you help me please?
Penny Nom lui répond.
The distance between overlapping circles 2012-07-26
Jeff pose la question :
I have two circles of different size that overlap one another: Circle #1 has an area(A) of 731,475, so I can calculate its radius as 482.6. Circle #2 has an area(A) of 502,517, so I can calculate its radius as 400. If I know that the area where they overlap is 179,271, how can I calculate the distance between the midpoints of these two circles?
Chris Fisher lui répond.
A truncated cone: the central angle 2012-07-17
Tom pose la question :
I have researched several sites, including this one and am fairly confident I can do the calculations required to produce the two radii and the slant height for a truncated cone. I a somewhat confused by the central angle. Some sites indicate that it should never exceed 180 degrees while others do not. Different examples on your site seem to use this in two different ways in constructing a truncated cone. Sometimes the angle seems to indicate the section to be removed, while in other examples the angle seems to indicate the section to be saved. Since the two angles will always total 360 I am confused about how to use the info to calculate the minimum rectangle required to contain the pattern. I'm guessing that in some cases I may need to use the chord or sagitta to determine the desired height and width of the material. Any help is greatly appreciated. Thank you. Tom
Penny Nom lui répond.
Translation of a graph 2012-06-02
Sean pose la question :
the information is thus: $f(x)=3(x+1)^2-2$ I wish to translate up 2 units and move 2 units right.
Penny Nom lui répond.
Using Newton's Method to find a root 2012-04-09
Nancy pose la question :
Use Newton's method to find the real root function, accurate to five decimal places

f(x) = x^5+2x^2+3

Penny Nom lui répond.
A lot with a slope 2012-03-30
Carlos pose la question :
I have a lot with a slope. On the right I need to bring it up 2', From that point to the left is 112' which at this point I need to bring it up 5' for it to be level . The distance front to back is 80'. What is the cubic yards of this area that I need to fill in.
Harley Weston lui répond.
Two overlapping circles 2012-03-21
Monty pose la question :
If you have a 3.75" radius circle overlapping a 5" radius circle with their centers 3" apart what would be the area of the non-overlapped portion of the small circle?
Penny Nom lui répond.
A trapezoidal prism filled with sand 2012-03-12
Maria pose la question :
could a Trapezoid that is 80 inches high with a 54 inch width top and a 89 inch width bottom and a 80 inch depth hold 58 cubic yards of sand?
Robert Dawson lui répond.
A water tank is in the shape of a truncated cone 2012-03-08
Victoria pose la question :
Suppose you have a water tank in the shape of a truncated cone. The larger diameter is K, the smaller diameter is K/2, and the height is also K (all measured in meters). The force of gravity on an object of mass, m, is F=9.8m (measured in Newtons). The density of the water is 1000 kilograms per cubic meter. What is the volume of the tank and how much mass will the tank have when it is full?
Penny Nom lui répond.
Wallpapering a room 2012-02-08
jimmy pose la question :
Ms. Frank is going to wallpaper a living room with dimensions 24 feet long, 18 feet wide, and 8 feet high. What surface area does Ms. Frank plan to wallpaper?
Penny Nom lui répond.
The graph of a quadratic function 2011-12-27
Thomas pose la question :
sketch a quadratic function with zeros at -3 and 1
Penny Nom lui répond.
2 + f(x) and f(x) + 2 2011-11-21
Beth pose la question :
Do you do y=2+f(x) the same way as if the 2 came after the equation such as y=f(x) +2 when graphing?
Penny Nom lui répond.
The end of a trough 2011-10-21
Lorraine pose la question :
The vertical end of a trough has the following dimensions: width at top 4.4 ft., width at bottom 3.2 ft., depth 3.5 ft. Find the area at the end of the trough.
Penny Nom lui répond.
One central circle and three tangent circles 2011-10-16
Margaret pose la question :
You have one central circle and three or more circles tangent to the outside of the circle of varying radii. You know the x,y coordinates of the centers of the other circles. If you now remove that central circle (and pretend you never knew where it was), can you calculate its center in x,y coordinates?
Chris Fisher lui répond.
A trapezoid inscribed in a circle 2011-10-02
Greg pose la question :
A trapezoid is inscribed within a circle. The two interior angles who share the longest side are 70 and 80. The arc whose chord is the longest side has a length of 120. Find the other two interior angles of the trapezoid, and the other three arc lengths.
Chris Fisher lui répond.
A step function 2011-08-22
Giovanna pose la question :
An average school bus holds 45 people. Show a graph between the relationship of students who need bus transportation and number of buses required?
Robert Dawson and Penny Nom lui répond.
A trapezoid 2011-08-02
Nazrul pose la question :
How can I draw a trapezoid whose parallel sides and other two sides are given.
Chris Fisher lui répond.
A stained glass lamp 2011-07-25
Guy pose la question :
Like Kay, I also work in stained glass, but in 3-dimensions. I am frequently asked to replicate lamp shades in stained glass where the diameter of the top is different (narrower) from the diameter of the bottom (which is wider). Some people want 5, 6, 7, 8, 10, 12, 16, ,,, n-sided shades. Is there a formula I can use to determine the width of the sides using the angle, if I remember correctly, I think it's called theta. In other words, is there a formula where I can plug in the angle which describes the arc of the circle. For instance, if someone asks for a 7 sided shade, plugging in 51.43 (360/7). I could then use that to determine the width at the top and bottom rings to create the appropriate trapezoids. I've visited a few sites so the formula looks like its a function of sin & cos but they are presented like proofs for teaching. Your site appears to want to actually answer questions without making the inquirer feel stupid.
Harley Weston lui répond.
A reel of paper 2011-06-03
keith pose la question :
if a full reel of paper has 7750 metres on it with a diameter of 1240mm and a core diameter of 100mm how can work out the thickness of each strip of paper therefore allowing me to work out the metreage of the roll when its half in diameter at say 620mm
Penny Nom lui répond.
The volume of a triangular shaped aquarium 2011-05-07
Jeff pose la question :
HI, I searched the site but really couldn't find my answer, at least that I could understand! I have a triangular shaped aquarium ( think of a cube, except its an equal triangle shape). all 3 sides are 48 inches, and it is 34" tall. Can you please tell me what the volume is in gallons? Thanks!
Penny Nom lui répond.
cos(x) = -1/(square root of 2) 2011-04-27
Shelby pose la question :
Find exact value of x for 1 <(or equal to) x < 2pi a) cos(x) = -1/(square root of 2)
Penny Nom lui répond.
Linear inequalities 2011-03-25
Alice pose la question :
OK. I'm in algebra 1 and the linear inequality is
y<2x-1
y>2
I don't completely know how to graph y>2 and I'm getting really frustrated
Please help me in any way possible!
Thank You!

Penny Nom lui répond.
6 faces and 8 corners 2011-03-04
noemi pose la question :
jeff made a figure with 6 faces the figure has 8 corners all of the faces are the same size and shape what figure did jeff make
Penny Nom lui répond.
At what rate is the grain pouring from the chute? 2011-02-26
MJ pose la question :
Suppose that grain pouring from a chute forms a conical heap in such a way that the height is always 2/3 the radius of the base. At the moment when the conical heap is 3 m high, its height is rising at the rate of 1/2 m/min. At what rate (in m^3/min) is the grain pouring from the chute?
Penny Nom lui répond.
A circle is inscribed inside an isosceles trapezoid 2011-02-25
priyam pose la question :
a circle is inscribed inside an isosceles trapezoid (with parallel sides of length 18 cm and 32 cm) touching all its four sides. find the diameter of the circle. thanks for help!!
Penny Nom lui répond.
Making a truncated cone 2011-02-18
lisa pose la question :
We need to make a cone that has the following dimensions.
14-3/8" diameter on large end
13-3/8" diameter on small end
4" tall

What are the dimensions I need to cut in order to make a cone with one seam?

Stephen La Rocque lui répond.
Points on a graph 2011-02-15
Maggie pose la question :
Which set of ordered pairs can be used to graph y = -2x-1.

A.(-5,9),(1,-3),(4,9)
B. (-2,-5),(1,-1),(5,9)
C.(-2,-5),(0,-1),(4,-3)
D. (-5,9),(-2,3),(4,-9)

I cannot figure it out. Could you help me?

Penny Nom lui répond.
Calibrating a conical tank 2011-02-05
Bill pose la question :
Hi, I have a round tank with tapered sides where I know the diameter at the top and bottom. Is there a formula I can use to calculate the volume by measuring from the bottom up the side (at the angle of the side) to any given point? Thanks, Bill
Stephen La Rocque and Penny Nom lui répond.
The height of the trapezoid 2010-12-01
Ryan pose la question :
A trapezoid has an area of 10.5 cm squared and parallel sides that measure 5.0 cm and 2.0 cm what is the height of the trapezoid
Penny Nom lui répond.
Intersecting polygons 2010-11-17
Laura pose la question :
How can I estimate the amount of intersection (area or better a normalized number) between two polygons if I have the cartesian coordinates of their vertices?

Thank you in advance,
regards,
Laura

Robert Dawson lui répond.
The sides of a trapezoid 2010-11-03
Bobbi pose la question :
Isosceles Trapezoid base=60”; two 45⁰angles at base, two 135⁰ angles at the top, vertical distance between base & top=16”. What is the length of the top line and the two equal side lines?
Penny Nom lui répond.
A water trough 2010-10-22
Jasmine pose la question :
A water trough is 8 m long and its cross-section is an isosceles trapezoid which is 90 cm wide at the bottom and 120 cm wide at the top, and the height is 30 cm. The trough is not full. Give an expression for V , the volume of water in the trough in cm^3, when the depth of the water is d cm.
Stephen La Rocque lui répond.
The equation of a line 2010-10-19
Sava pose la question :
I have to write an equation and sketch a graph for the line that meets the given conditions:

A line with slope -15/5 that passes through the point (-2.5,4.5)

Penny Nom lui répond.
A taxi ride 2010-09-22
Jack pose la question :
A taxi ride costs you $2 for the first km and 10 cents for each tenth of a km thereafter. Sketch a graph of the total cost $C(x) for a ride of xkm, for 0 < x < 2. Hint: The graph is not smooth.

How will I start this question off? I tried the table of values method, and then graphed it, but it comes out as smooth,

Stephen La Rocque lui répond.
How many labels are left on a roll? 2010-08-13
Melissa pose la question :
Is there a simple way to calculate how many labels are left on a roll? I thought there was a way to look at the individual label length, the label thickness and the core diameter and then measure the overall diameter to calculate how many pieces were left, but I'm struggling. I've seen a tool before- I thought we called it a circle calculator, but those parameters were all that I needed to input to spit out the correct count. I never saw what was behind the scenes for this calculation. Ie factoring in revolutions or the number of pieces on each layer, etc. Please let me know if you have any suggestions. Any assistance will be greatly appreciated!!! Thanks!
Penny Nom and Tyler Wood lui répond.
The intersection of two graphs 2010-06-27
Austin pose la question :
In depth explanation of how to find the intersection points of functions y=-4t+300 and y=-4.9(t-5)^2+300
Stephen La Rocque lui répond.
The outside surface area of a U shaped tube 2010-06-03
Adolfo pose la question :
What is the formula for calculating the outside surface area of a U shaped tube?
Robert Dawson lui répond.
graph y=(2x^2-3x)e^ax 2010-05-31
James pose la question :
graph y=(2x^2-3x)e^ax
Robert Dawson lui répond.
Which values of x satisfy (x-2) / (x+4) <7 2010-05-23
Rocco pose la question :
Which values of x satisfy (x-2) / (x+4) <7
Harley Weston lui répond.
Two overlapping circles 2010-04-12
Scott pose la question :
There are two circles, big circle with radius R and small one with radius r. They intersect and overlap in such a way that the common area formed is 1/2 pi r^2 (half the area of the small circle). The Question is: suppose we have known the radius r of the small circle, and the distance between the two circle centers, what should the radius R of the large circle be?
Chris Fisher lui répond.
Vapor trails 2010-04-12
Frank pose la question :
I'm not sure if this is a proper question to ask so if I have misdirected my question I apologize and no response is expected. I am trying to figure out a way to measure vapor trails from my back yard in Phoenix Arizona. If I used a compass and spread each point of the compass to the start and finish of the vapor trail I would have the angle of an isosceles triangle. The other two angles would be identical. The height of from the inverted base of the triangle to my standing spot on the ground would be about 35,000 feet. I'm thinking that there should be a way to figure out the length of the inverted base (vapor trail) but I'm devoid of mathematical skills and can't seem to figure out how to do this. Is it possible to figure out the length of a vapor trail using this method or do you have an easier way to accomplish the task?

Any help you could offer would be most appreciated.

Thanks....Frank

Harley Weston lui répond.
An isosceles trapezoid is inscribed in a circle 2010-04-06
Abby pose la question :
An isosceles trapezoid whose bases have lengths 12 and 16 is inscribed in a circle of radius 10. The center of the circle lies in the interior of the trapezoid. Find the area of the trapezoid
Penny Nom lui répond.
A roll of paper 2010-03-25
keith pose la question :
how many yds of paper for a roll with a width of 6.875 and a core of 3 in and a diameter of 40in?
Robert Dawson lui répond.
Modelling the roof of a house 2010-03-23
Sandi pose la question :
This question has been haunting my dreams at night. It states the cross section of the roof of a house is modelled by the function y= -5\12|x-12|+5, where y>0 or y. Now I'm suppose to graph the function. b) Find the slope, height, length of sides, and base of the isosceles triangle. c)Explain what transformation must be applied to the graph of y=|x| to obtain the graph of y= -5\12|x-12|+5, where y>0 or y=0. 1st I put the equation into my calculator and graphed it. I got 10 for the height. We have had 2 sub teachers this past week. The first one told me the height should be 12 the other one told me 5. So I'm really confused. If I use my 10 units as height and the Pythagorean Theorem I get my sides to be 26 units the base 48 and my slope I calc to be 5/12. Both teachers are pretty sure I'm wrong and they are right but we all have different answers. Please help put this problem to rest so I can rest soundly at night. Thank you for your time.
Penny Nom lui répond.
A quadratic equation 2010-03-15
rachelle pose la question :
under certain condition, it is found that the cost of operating an automobile as a function of speed is approximated by a quadratic equation. use the data shown below to find the function. then use the function to determine the cost of operating the automobile at 60mph.
speed
(in miles per hour)
operating cost
per mile(in cents)
10 22
20 20
50 20

Penny Nom lui répond.
Two overlapping circles 2010-03-07
Hayden pose la question :
I have two circles of equal size. The radiuses of the circles are 30ft. The two circles are positioned 40ft apart and I need to find the area where they overlap.
Harley Weston and Tyler Wood lui répond.
Sketch the graphs of the following 2010-03-02
musaf pose la question :
without detailed plotting of points,sketch the graphs of the following showing relevant information on the graphs:
a) y=(x-3)2 +5
b)y=4x-x2

Penny Nom lui répond.
The area of a trapezoid 2010-03-01
mariah pose la question :
the height of a trapezoid is 6 centimeters. one of the bases is three times the height, and other base is four times the height . find the area of the trapezoid
Penny Nom lui répond.
A paper tube 2010-02-21
shailesh pose la question :
How can i estimate weight of paper tube from size. i have paper tube inner diameter : 22mm, thickness : 2mm, length : 14mm. so how can i got weight of paper tube?
Harley Weston and Tyler Wood lui répond.
Bending a wire to form shapes 2010-02-04
Geraldine pose la question :
a wire bent into the shape of a square encloses an area of 25cm squared. then the same wire is cut and bent into two identical circles. what is the radius of one of the circles round to the nearest hundred
Penny Nom lui répond.
A 3D cardboard cupcake 2010-01-30
Margaret pose la question :
Hi,
I'm an art student and I'm attempting to build a 3D cupcake out of cardboard. I want it to have a circular base of 8.5 inches and sides that are 7'' tall and slope outward so the top of the base is 29'. The top with the frosting would be a detachable lid made from a cone with a base circumfrance of 29". My problem is how to cut the side so they will slope out, I'm pretty sure there needs to be a curve, however I don't how to calculate the degree of it. Ideally I want the sides to be a single piece of cardboard. I also don't know how to calculate the arc needed to make a cone who's circular base's circumfrence is 29''.
Thanks,
Margaret

Stephen La Rocque lui répond.
The adjacency matrix of an undirected graph 2010-01-15
Bhavya pose la question :
Let Cn be the undirected graph with vertex set V = {1,2,3,...,n} and edge set E = {(1,2), (2,3), (3,4),.... , (n-1,n), (n,1)}. Let An be the adjacency matrix of Cn.
a. Find the determinant of An.
b. Find (An)^2

Robert Dawson lui répond.
The area of a trapezoid 2010-01-06
maureen pose la question :
the question asks for the area of a trapezoid with parallel sides 5m and 6m and height of 2.5m I have done it by dividing the trapezoid into 2 triangles and reach the same answer as my manual---11.25 sq. m My question...how would you illustrate this? I read this to mean to sets of parallel sides, which would mean it is a parallelogram and not a trapezoid. Any suggestions?
Robert Dawson lui répond.
Cutting a square into four shapes 2010-01-05
Tracy pose la question :
A square with a side of 4 ft needs to be cut into four shapes with the same area. can you find six different was to cut the square?
Robert Dawson lui répond.
Using graphs 2010-01-04
Armstrong pose la question :
Given that 50 litres of kerosine costs 6000 nira. Draw a graph and read off;
A: The costs of 15 and 34 liters.
B: The number of litres that can be bought for 2300 nira.

Robert Dawson lui répond.
A Lagoon, free form, inground swimming pool 2009-12-22
donna pose la question :
What is the linear footage of a 14 x 23, Lagoon, free form, inground swimming pool?
Robert Dawson lui répond.
Figure out the square root of something 2009-12-20
eric pose la question :
is there an ez equation to figure out the square root of something?
Harley Weston lui répond.
The acute angle formed by intersecting lines 2009-12-17
Katie pose la question :
To the nearest tenth of a degree, find the size of the acute angle formed by the intersecting lines 3x + 2y = 12 and x - 2y = -2. Was is necessary to find the intersection point?
Robert Dawson lui répond.
f(x)=x+2sinx 2009-12-12
amroziz pose la question :
for which values of x does the graph of f(x)=x+2sinx have horizontal tangent
Harley Weston lui répond.
The graph of f(x)=1/x -2 2009-11-29
Kapilan pose la question :
Please answer the following question:
Sketch the graph of f(x)=1/x -2

Harley Weston lui répond.
Two overlapping circles 2009-11-19
Raraa pose la question :
There are two identical circles . The edge of one circle is at the middle point of the other circle. There were overlapped . The area of the overlapped surface is 20000 square centimetres . How do I find the radius of the circle rounded to the nearest whole centimetre ?
Penny Nom lui répond.
The graph of f(x-2) 2009-11-19
Kapilan pose la question :
Hi please answer the following question:
Given the function f(x)=x^2 sketch the graph of f(x-2)

Robert Dawson lui répond.
An isosceles trapezoid 2009-11-12
lyjah pose la question :
what is parallel sides of an isosceles trapezoid measure 5cm and 11cm long.and oneof the other sides also measures s5cn long what of the isosceles trapeziod
Penny Nom lui répond.
A truncated cone 2009-11-11
Lucian pose la question :
I need to calculate the bottom inside diameter of a truncated cone.
The top insdie diameter is 1450mm.
The material is 6mm thick
The cone angle is 20 degrees
The slant length is 152mm
I would like a formula so that I can build a spread sheet

Penny Nom lui répond.
f(x)= (e^x) / [(e^x)+(ex^2)] 2009-11-10
natalie pose la question :
I'm trying to graph the function, f(x)= (e^x) / (e^x)+(ex^2) [e to the x divided by e to the x plus e times x squared] I know that there aren't any vertical asymptotes, but is there a horizontal asymptote? and also, I'm stuck on finding the concavity for this graph. I tried to find f "(x), but it came out to be really long and I am not sure how to find the x values for f "(x) without using a graphic calculator. thanks, natalie
Chris Fisher and Harley Weston lui répond.
The intersection of two graphs 2009-10-30
asma pose la question :
by using the suitable scales,draw both the graph of x/2 + y/4 =3 and y=2 on the same graph paper.Find the points of intersection of the two graph.
Penny Nom lui répond.
Graphing y=(4-x^2)^5 2009-10-25
natalie pose la question :
I want to graph the curve of y=(4-x^2)^5 without using a graphing calculator. To do this, I'm suppose to find: domain, y and x intercepts, asymptotes, intervals of increase/decrease, local max/min, concavity and points of inflection. I got all the way to the step where I'm solving the concavity and I'm stuck. I found the f"(x) and it came out to be really large polynomial. I want to know how I can solve for the x of f"(x) without the use of a graphing calculator, when the polynomial has x^6 and x^8. Thank you so much, natalie
Harley Weston lui répond.
The intersection of two graphs 2009-09-02
Jaqueline pose la question :
Find the points of intersection of the graphs of the equations

x^(2 )+ y^2=25 and 2x + y = 10

Penny Nom lui répond.
y=2x+1 and y=2x-1 2009-08-28
MARICELA pose la question :
Need help of how to work this problems

y=2x+1 and y=2x-1

First of all what is the difference?

Second I think is easy just to work with the X y with the line in the middle

but how do you get the numbers for each side.

Penny Nom lui répond.
Graph 2x-3y+15=0 2009-08-20
Diem pose la question :
Graph 2x-3y+15=0
Leeanne Boehm lui répond.
A paper towel roll 2009-08-19
Jeff pose la question :
I am making a spiral tube with paper that is 2" in dia. and 102" long I will be using paper that is slit 3" wide how many lineal feet of paper will I need to to cover the 102" I will be using 3 rolls of paper that will over lap the other by half to make a hard tube (paper core) in a roll of paper towels Thanks Jeff
Penny Nom lui répond.
The volume of a telephone pole 2009-08-11
robert pose la question :
12.5"@ base x 7" @ top and 40' height. How would I find the volume in cubic feet for a telephone pole with these dimensions?
Harley Weston lui répond.
The volume of a tapered pole 2009-08-09
robert pose la question :
what is the formula to use to come up with cubic feet in a tapered pole
Harley Weston lui répond.
The length of tape on a reel 2009-08-06
Philiip pose la question :
What diameter reel would be required to roll up 1000 meters of tape that is 0.01 mm thick such that the final role left 10 mm empty space between the top of the roll and the outside of the reel, the hup that the tape was wound on was 2 cm?
Stephen La Rocque lui répond.
Buying pens and staplers 2009-07-22
Ellen pose la question :
one more math question,
six pens and four staplers cost $7.10. four pens and seven staplers cost $9.50 find the cost of one pen and one stapler.

Penny Nom lui répond.
A roll of paper 2009-07-05
mark pose la question :
is there a simple way of finding out how much is left on a roll of paper. i have read similiar questions and answers on here but all seem very complicated and not being very good at maths does not help me much. some are in inches and all have different figures to mine so if i give my figures hopefully i wil understand it better. the radius of the cardboard core is 52.25mm, the radius of the paper at 2000 linear meters is 158.625mm. the thickness is 0.17mm. when nearing the end of the paper how do i work out how much is left in a simple way. thank you in advance for any help you can give me
Harley Weston lui répond.
The surface area of a tank 2009-06-25
Charles pose la question :
I need to calculate the external wall and top sqft surface area of a tank 54ft od x 15ft high with a dome top 54ft od x 8ft high.
Penny Nom lui répond.
How many times does the graph of y=2xsquared -2x + 3 intersect the x-axis? 2009-05-27
Henry pose la question :
how many times does the graph of y=2xsquared -2x + 3 intersect the x-axis?
Penny Nom lui répond.
The Smythe family lot 2009-05-24
Shrima pose la question :
We must determine the size of the lot. The Smythe family lot is 235ft across the front and 212ft down the side and 254ft down the other side. The sides of the lot are parallel to each other and perpendicular to the front. The size of the lot is two dimensional measure therefore you need to find the areaenclosed by the lot line.
Question:

What is the size of the Smythe lot including the unit?

Penny Nom lui répond.
The dimensions of a larger cup 2009-05-23
Elizabeth pose la question :
Hey If i have a cup that holds a volume of 477mL and the bottom radius is 2.8cm and the top radius is 4.9cm and the height is 10cm. If i increase the volume by one and a half times what is the new measurements if the cup is directly proportional to the first one. Thank you
Stephen La Rocque and Penny Nom lui répond.
Stairway construction 2009-05-02
Olivia pose la question :
Hi, I'm Olivia and I'm in 8th grade studying algebra 1. I'm having some trouble with my math project. It has to do with rules of stairway construction. It says that there are two generally accepted rules to building stairs. Rule 1: x+y=43 and x+y=46 (cm) Rule 2: 2y+x=61 and 2y+x=64 (y is the riser and x is the tread length) I cant figure out how to graph them...it says they're supposed to intersect to make a square which is like the safety zone for building stairs (the stairs' rise and tread lengths are plotted as points) any points outside of that square are considered unsafe. Did i explain it ok? Thanks for your help. --olivia
Penny Nom lui répond.
Winding paper after a break 2009-04-10
Olen pose la question :
Question from Olen:

I work in a paper mill and have been handed the task to search for a formula to determine how much paper needs to be added to a parent roll to make up the difference at the winder. (Ex. The spool diameter at the reel is 18.25" we measure roughly 33.5" to make two 58" rolls in the winder. If the is a paper break and the roll diameter in the winder is 30" how much do I add to a single parent roll (22" roughly) to make one 58 " and the 28" needed at the winder. I would appreciate any help to complete this task. I would like to be able to build a chart that operators can refer to based on what is needed. Thank you.

Harley Weston lui répond.
The graph of y = 4x^-4 2009-04-07
lyric pose la question :
tell whether the graph opens up or down.write an equation of the axis of symmetry
y=4x^-4

Penny Nom lui répond.
A triangle inside a trapezoid 2009-04-06
rohan pose la question :
i have a diagram with a triangle inside a trapezoid.trapezoid is PQRS , PQ being the line above and RS THE LINE BELOW.The area of this is 12. If RS is twice PQ what is the area of PQS.
Penny Nom lui répond.
Conversion factor 2009-03-23
kab pose la question :
Would like to photocopy a Landscape Design. What is the Conversion Ratio or magnification required to convert 1" = 10 feet to 1/4" = 1 foot?
Harley Weston lui répond.
A 1.5:1 rectangle 2009-03-23
melody pose la question :
I have an origami model that says to use a 1.5:1 rectangle. I don't understand how to calculate the size of paper I need to use. Surely there is a formula to calculate this ratio? Thanks for your help.
Penny Nom lui répond.
A simple digraph 2009-03-23
Shabkhal pose la question :
Show that the sum of in degrees of all the nodes of a simple digraph is equal to the sum of out degrees of all its nodes and this sum is equal to the number of edges of the graph.
Victoria West lui répond.
An equation for a line with a changing slope 2009-03-21
Ben pose la question :
Is there anyway to write an equation for a line with a changing slope. e.g. A cell [hone company has a initial fee of 50$ paying for the first 100 minutes, the rate then increases to 10 cents per minute, then at 500 minutes the rate is 5 cents per minute.

If so please tell how.
Thanks

Penny Nom lui répond.
A property that decreases in value every year by 10 percent 2009-03-07
PEter pose la question :
there is a competition problem where a cottage is worth 25000 years, but it decreases in value every year by 10 percent. Without a calculator, is there a way to find after how many years, the cottage will be worth 2500? (besides multiplying it out and besides logs)
Stephen La Rocque lui répond.
A shape with 10 sides 2009-03-03
Sandra pose la question :
One of my son's math questions is as follows: Suppose I have a shape with 10 sides. I choose a vertex then draw lines to the other vertices that dont share sides with the first vertex. How many vertices will that be and why?
Penny Nom lui répond.
An octagonal landscaping frame 2009-03-01
Richard pose la question :
Hi I am trying to put landscape timbers down in octagon shape that measures 6 feet across and outside measures 360 degrees.. The timbers are 4 inches by 4 inches. I need to know at what angle to cut boards and at what length i need to complete octagon.
Thanking you in advance for your kind assistance.
Richard :)

Harley Weston lui répond.
The graph of y = -6 2009-02-21
Barb pose la question :
I was given the problem that says "graph the line y = -6" how do I find all the set points that have a y coordinate of -6? I am so lost lost and have no clue how to start. Can you help me please? thanks.
Penny Nom lui répond.
Making shapes with straws 2009-02-06
Melissa pose la question :
suppose you have 60 straws. How many of each shape could you make
a) Triangles
b) squares
c) pentagons
d) hexagons

Stephen La Rocque lui répond.
The intersection of two lines 2009-01-30
kaylee pose la question :
how do you find and graph the intersect of these two equations: y=2x-4 (and) 2y=x thanks, kaylee
Stephen La Rocque lui répond.
Shapes 2009-01-29
Rana pose la question :
I want to know about Circumference / Area / Volume of different shapes ie Circle , Oval , An Arc Trapezide etc
Robert Dawson and Harley Weston lui répond.
y = |x| - 4 2009-01-28
tiana pose la question :
how do you find where to plot Y in a problem that states y=|x| -4?
Robert Dawson and Penny Nom lui répond.
Graphing a parabola 2009-01-27
Kimberly pose la question :
I need help with this parabola: graph y=5x^2-5x-6. I am not really understanding how to graph parabolas in general. Can you help me?
Penny Nom lui répond.
A applied math trig problem 2009-01-13
Simon pose la question :
I wish to find all the answers for the following equation over the interval (0,1):
cos^2(pi * n^x) + cos^2(pi * n^(1-x)) - 2 = 0 where n is any integer > 0

Robert Dawson and Harley Weston lui répond.
Surface area of an irregular shape 2008-12-15
Patrick pose la question :
An irregular shaped object (lets say a gold nugget, not smooth with pockets) can have its volume determined by comparing its mass in water.

Is there any method or means or anything that could be used to determine the surface area of this shape? Whether that be theoretical mathematical formula to using a special infrared technique,etc...

The problem I foresee is that the component parts cannot be divided into smaller geometric shapes. I would propose an answer although I don't know if it is a good one: A liquid material that dries super-thin, but has a very specific and easily determined volume/mass is coated over the object. Measure the mass difference between the beginning sample of fluid and the mass after the object has been coated. Then determine the surface area of the same mass of fluid in a geometric shape. Is this feasible?

Robert Dawson lui répond.
Wall covering 2008-12-10
Jeanie pose la question :
I need to convert 14 LY of wallcovering (54" wide) to square feet. Can you please help me?
Penny Nom lui répond.
A quadrilateral with exactly 1 pair of parallel sides and no congruent sides 2008-12-10
alie pose la question :
a quadrilateral with exactly 1 pair of parallel sides and no congruent sides is what?
Robert Dawson lui répond.
Graphing 2008-12-09
Natasha pose la question :
2x+y=5
Robert Dawson lui répond.
Graphing Rational Functions 2008-12-05
Bahra pose la question :
The question in my homework is: Find all vertical, horizontal, and slant asymptotes, x- and y-intercepts, and symetries, and then graph each function. Check your work with a graphing calculator. y=3/x+3 meanng: ( y= 3 over x+3)
Janice Cotcher lui répond.
Evaporation of waste water 2008-11-21
Alexandra pose la question :
Engineers know that the amount of water lost due to evaporation is directly proportional to the surface area of the holding tanks and that the local climate causes water to evaporate at the rate of about gallon of water per hour for each square foot of water exposed to the air. About how many total gallons of waste water may enter the three tanks each day to maintain full capacity?
I know that the answer is 50x10^3 but i don't know why.

Penny Nom lui répond.
Four circles in a square 2008-11-19
Anthony pose la question :
I have a square where one side measures 10cm and within that square There are four equal quarter circles. Each quarter circles starts in a different corner of the square and I am trying to find area inside the overlap on the quarter circles.
Janice Cotcher, Chris Fisher and Penny Nom lui répond.
The diagonals of an isosceles trapezoid 2008-11-14
hazel pose la question :
how to solve the diagonals of an isosceles trapezoid? what is the formula?
Harley Weston lui répond.
Filling a tank with 2 taps 2008-11-10
Murray pose la question :
2 taps turned on together can fill a tank in 15 minutes. By themselves, one takes 16 minutes longer than the other to fill the tank.Find the time taken to fill the tank by each tap on it's own.
Penny Nom and Victoria West lui répond.
The area of a trapezoid 2008-11-04
kelsey pose la question :
The area of a trapezoid is approximately 150cm2...the length is 26 cm, the weight is 5cm...what is the height????
(show me the work that you did so i can understand)

Penny Nom lui répond.
The volume of a pond 2008-11-04
Malcolm pose la question :
I am looking for a formula to find the volume of my pond. I know the average surface diameter and the average bottom arc length.(The arc from one bank through a point in the bottom and on to the surface on the other bank. I am not able to get a good measurement of the depth at this time. Can the volume be figured with the known measurements (the cord and the arc)? Thanks: MS
Harley Weston lui répond.
Scaling a logo 2008-10-20
Carl pose la question :
Hello! I hope that you can help me out here. I've been trying to figure this out for awhile now and can't come up with the answer logically. I am a graphic designer by trade and need to figure out how large my client's logo should be. The logo will need to be sized proportionally with the surface area of the page. The logo will always be sized at 1.2938(w) x .75(h) on a letter size page (8.5 x 11).
My question is...
When you increase the size of the page (not always proportionally sized - dimensionally) how do figure out the NEW size of the logo. I could have a page 20" x 40" or 30' x 4'... or ANY size really. I believe I have the new surface area of the logo but can't find the new dimensions.

Harley Weston lui répond.
Concavity and the second derivative 2008-10-15
Christina pose la question :
I'm having trouble solving for a second derivative for the following graphing question.

f(x) = (X^2+2x+4)/2x

using the quotient rule, I found:
f'(x) = (x^2-4)/(2x^2)

however, using the quotient rule again I can't seem to solve it (concavity):
f'''(x)=[(2x)(2x^2)-(x^2-4)(4x)]/[(2x^2)^2]
f''(x)=[(4x^3-(4x^3 -16x)]/4x^4
f''(x)=16x/4x^4
f''(x)=4/x^3

and making the equation equal to zero result in 0=4 which doesn't seem to make sense...

Penny Nom lui répond.
Compatible numbers 2008-09-25
Katie pose la question :
I am having trouble remembering how to find compatible numbers used to find the estimate in the foloowing problem. Can you please help me solve it and explain it to me? My son is in 5th grade and I need help. Thank you. Here is the problem: 2,752 / 28 estimate: 90
Penny Nom lui répond.
Graphing a piecewise function 2008-08-12
Shanti pose la question :
how do you graph a step function or piecewise function such as
u^2 if u is greater than or equal to -2 and less than 1
2- u if u is greater than or equal to 1 and less than 4

Penny Nom lui répond.
Two equations in two unknowns 2008-08-01
James pose la question :
Question from james, a student:

3x+2y=9;24x-2y=18

My math teacher says that it is (2,-4)

Can someone please explain how to figure this out by graphing?

Victoria West lui répond.
The cube root of a really big number 2008-07-17
Pete pose la question :
Is the only way to find a cube root of a really big number guess and check, as i read in other questions, or is there another way without a calculator?
Harley Weston lui répond.
Graphing Using Double Angle Identities 2008-07-16
Hodan pose la question :
the Question is: Describe how you could use your knowledge of Double angle formulas to sketch the graph of each function. Include a sketch with your description.A) F(x)=sin x cos x B)F(x)=2 cos(squared)x C) F(x)= tan(x) (divided) by 1-tan(squared) x
Janice Cotcher lui répond.
Domain & Range of a Periodic Function 2008-07-11
Michelle pose la question :
The depth, w metres, of water in a lake can be modelled by the function, w=5sin(31.5n+63) +12 where n is the number of months since January 1, 1995. Identify and explain the restrictions on the domain and range of this function.
Janice Cotcher lui répond.
A point on a graph 2008-07-08
Rita pose la question :
If (2, b) is a point on the graph of y = (x^2) + 4x, find the value of b.
Leeanne Boehm lui répond.
Points on a graph 2008-07-03
Rita pose la question :
Dertermine if the given 3 points are on the graph of the equation.

Equation: y = (x^3) - [2(sqrt{x})]

Points: (0, 0); (1, 1); (1, -1)

Penny Nom lui répond.
A piecewise equation 2008-06-16
John pose la question :
AT&T charges $39.99 for 450 minutes per month. It charges 45 cents per minute over the 450 minutes. How can I make a piecewise equation of this problem?
Penny Nom lui répond.
How do i find the area of an irregular figure? 2008-06-10
DAVE pose la question :
How do i find the area of an irregular figure?
Penny Nom lui répond.
Geometric shapes that are the same 2008-06-09
tia pose la question :
what is the name used for shapes that are exactly the same?
Harley Weston lui répond.
The area of a regular pentagon 2008-06-02
Jenna pose la question :
I would like to know if there is a formula to find the area of a regular Pentagon that is not inscribed in a circle only apothem and side length are given.
Penny Nom lui répond.
Applications of trigonometry 2008-05-24
Mohita pose la question :
I have got a project in the school and i am not getting anything about the topic. The topic is that we need to find the application of trigonometry on any one of the real life situations using 3-dimensional figures. I mean how can trigonometry can be used in real life situations like navigation, architecture, survey, astronomy etc.
Penny Nom lui répond.
The wedges in a circle graph 2008-05-18
Libby pose la question :
I don't understand how to do this word problem: The cafeteria workers distributed a survey to the student body asking students to pick 1 from a list of 5 choices for their favorite lunch. The circle graph below gives the results of the survey for the students who responded. What is the measure, in degrees, of the central angle for each choice?

And there is a pie chart that has food choices with these percentages: 25%, 20%, 15%, 10%, and 30%

Leeanne Boehm and Penny Nom lui répond.
The area of a trapezoid 2008-05-15
Ashley pose la question :
what is the formula for area of a trapezoid
Penny Nom lui répond.
The fourth side of a property 2008-05-14
Penny pose la question :
Property has 3 measurements and 2 angles and I want the altitude at a particular point. Corners are sw, nw, ne and se. 1 want the 4th saide measurement confirmed (we think 146.07') and the altitude measurement at 25' from sw (on the sw-se side) The angles provided are interior at nw 67.38 degrees and the opposing corner se interior appears to be 68.58 degrees. the 3 sides with measurements are: sw to nw=54.07'. sw to se 146.07' and se to ne 53.57'. If you would help, we would really appreciate it. Thanks! Penny
Harley Weston lui répond.
Identify the graph 2008-05-05
A parent pose la question :
Refer to the graph given below and identify the graph that represents the corresponding function. Justify your answer.
y = 2x
y = log2x
I can figure out what line on the graph is which equation but unsure what is meant by justify your answer.

Penny Nom lui répond.
Two overlapping circles 2008-04-26
Michelle pose la question :
Two overlapping circles O and Q have the common chord AB (vertical line between the overlapping circles). If AB is 6 and circle O has a radius of length 4 (horizontal line going through the overlapping circles and touching the side of the circle) and circle Q has a radius of length 6, how long is OQ.
Penny Nom and Walter Whiteley lui répond.
What shape(3-D) has 5 vertices,8 edges,and 5 faces? 2008-04-25
stacy pose la question :
what shape(3-D) has 5 vertices,8 edges,and 5 faces?
Penny Nom lui répond.
Naming a figure 2008-04-24
P. pose la question :
What is the name of a figure that has 2 sides that measure 25 feet each and 2 sides that measure 10 feet each?
Penny Nom lui répond.
A solid figure with 3 rectangular faces and 2 triangular faces 2008-04-17
Emily pose la question :
I am in the 3rd grade and my teacher gave us a question. What is a solid figure with 3 rectangular faces and 2 triangular faces?
Penny Nom lui répond.
The capacity of a silo 2008-04-12
Candonn pose la question :
I have a cement silo that is 30 ft. high and has a diameter of 12 ft. I was wondering what the capacity of the silo is in tons of dry material. The silo is flat on the bottom also.
Penny Nom lui répond.
A shape 2008-04-03
Joshua pose la question :
what shape has 4 faces, 4 corners and 9 edges.
Penny Nom lui répond.
The square root of 21 between what two numbers? 2008-04-03
Danielle pose la question :
The square root of 21 between what two numbers?
Penny Nom lui répond.
The cube root of 198 2008-03-27
bobby pose la question :
what is the cubed root of 198
Harley Weston lui répond.
How would I graph: x - 2 divided by x^2 - 4? 2008-03-19
Sean pose la question :
How would I graph: x - 2 divided by x^2 - 4 using a method in extreme values of functions or completing the square

x - 2/ x^2-4

Stephen La Rocque lui répond.
Graph the equation P(x)= x^3 + x^2 -12x 2008-03-18
Sean pose la question :
I want to graph the equation : P(x)= x^3 + x^2 -12x I was thinking that I would combine the x-values, but then I could not use the complete the square technique. Would I have to just solve the second half of the equation and then graph x^3 seperately
Penny Nom lui répond.
Determine the capacity of Farmer John's silo 2008-03-16
paula pose la question :
Farmer John stores grain in a large silo located at the edge of his farm. The cylinder-shaped silo has one flat, rectangular face that rests against the side of his barn. The height of the silo is 30 feet and the face resting against the barn is 10 feet wide. If the barn is approximately 5 feet from the center of the silo, determine the capacity of Farmer John’s silo in cubic feet of grain.
Penny Nom lui répond.
Estimate the cube root of 270 2008-03-11
Tom pose la question :
(a) By plotting suitable graphs, estimate to one decimal place the cube root of 270.
(b) With reference to your answer to part (a), use 8 iterations of a Bisection Search to refine your estimate. Use the nearest whole numbers either side of your estimate from part (a) as starting values.
(c) Using either of your starting values from part (b) as first guess, use the Newton-Raphson method to find the true value of the root (to 6 decimal places). Repeat using the other starting value from part (b) and compare the two results.

Harley Weston lui répond.
An incorrect Babylonian formula 2008-03-10
Martin pose la question :
Given the area of an isosceles trapezoid is 150, that the difference of its bases is 5 (that is b1 - b2 = 5), and that its equal sides are 10 greater than two-thirds of the sum of its bases (that is, s1 = s2 = 2/3(b1 +b2) + 10 Solve the problem using an incorrect Babylonian formula for the area of a trapezoid A = (b1 + b2)/2 x (s1 + s2)/2 Find the length of the sides of the isosceles trapezoid.
Penny Nom lui répond.
A roll of paper 2008-03-05
Sheik pose la question :
How I Convert My Roll Weight Into Meters?
Roll Width : 241mm
Thickness : 56gsm
Weight :81 Kg

Harley Weston lui répond.
Pythagorean equation 2008-02-27
Judy pose la question :
X^2 + Y^2 = 5^2
Graph all ordered pairs in the coordiante plane that satisfy the Pythagorean equation, x squared plus y squared equals five squared

Stephen La Rocque lui répond.
How do I graph y = -2x^2? 2008-02-12
Max pose la question :
How do I graph y=-2x^2?
Penny Nom lui répond.
A three sided shape 2008-02-09
Tim pose la question :
Question from Tim:

How would I calculate the area in square feet of a "triangle" with sides:

Base is 36 feet; height is 70 feet; and the thirs side is an arch (curve) of 77 feet.

Harley Weston lui répond.
Out of school applications of Pythagoras Theorem 2008-01-23
Laura pose la question :
Hi, I am currently working on a math summative in which I have to choose a real life subject and relate it back to the material in my grade 12 math class. I find the history and discovery behind the Pythagorean Theorem and Identity very interesting, but I have yet to find a real-life application of the equations. Yes, I know they are used for finding distances, heights etc., but realistically, how many people actually use it in those situations? Very few. I was hoping for a new application. Is the pythagorean theorem (sin^2x + cos^2x = 1) even applicable? Thank you, Laura
Harley Weston lui répond.
Graphing a parabola 2008-01-16
Sean pose la question :
How do I find the roots and describe the roots when graphing a parabola?

y = - x^2 - 4x -3

Stephen La Rocque lui répond.
Pie graphs 2008-01-15
amarilis pose la question :
explain why circle graphs are called pie graph or pie charts
Stephen La Rocque lui répond.
Hoisting an anchor 2008-01-10
ron pose la question :
four men on a boat are using a capstan for hoisting an anchor weighing 2000lbs. The diameter of the barrel of the capstan at the section in use is 8in., and the pull on the chain carrying the anchor and passing around the capstan barrel is assumed to act at a distance of 1in. from the surface of the barrel. If the distance from the axis of the barrel to each point at which effort is applied to a capstan bar is 40in., what force must be applied by each man hoisting the anchor?
Stephen La Rocque lui répond.
Practical applications of sequences 2008-01-01
carl pose la question :
can you give me examples of different kinds of practical applications of sequences?
Penny Nom lui répond.
Finding the equation of a line from its graph 2007-12-25
ken pose la question :
I have a graph and it is asking me to find the slope of each line, and then write its equation. the graph show a negative slope. Would appreciate the answer if you could show me how to do this question.
Kind regards: Ken

Stephen La Rocque lui répond.
Solve sin(x)=x^2-x 2007-12-11
ming pose la question :
is there anyway you can solve
sin(x)=x^2-x without a calculator?

Stephen La Rocque lui répond.
Pictographs 2007-12-11
eniyah pose la question :
The title of my pictograph is Favorite pets. I have two columns. in my first column is the kind of pet and the second column is the numbers of students, the dogs have 16 student that like them and cats have ten students the likes them. each pictures equals 2 stodents my question is. i have to draw paictures to show the number of students for each row. the key says each symbol equals 2 students. since 16 divided by 2 is 8 can you show me how to make this in a graph
Stephen La Rocque lui répond.
Plotting a function 2007-12-06
Paula pose la question :
My son has a problem that says plot the point: 2x-3y=9
I have explained to him that there must be a function for y in order to plot that point, but he doesn't think so. The question above that one says find the function for y, and I assume we should use that function. Am I wrong? Can we plot a point without a y function.

Stephen La Rocque lui répond.
A mystery shape 2007-11-26
Daniel pose la question :
I am in year 3 and have been given some homework to do with my mum. Question is: my shape has two circular faces and one curved face. It has two edges and no vertices. Can you help me identify the shape please.
Penny Nom lui répond.
A curve sketch 2007-11-22
Ahson pose la question :
Find critical points, determine the monotonicity and concavity and sketch a graph of f(x) with any local maximum, local minimum and inflection points labeled:

1. f(x) = x^4 - x^3 - 3x^2 + 1

Harley Weston lui répond.
Two lines 2007-11-18
Kate pose la question :
represent qraphically the equations of these lines and their intersecting points

x-y=2 3x+y=11

Penny Nom lui répond.
The area of a trapezium (trapezoid) 2007-11-05
Scott pose la question :
How can we calculate this I do not understand? A trapezium of area 126m2 has parallel sides of length 3m and 4m. How far apart are the parallel sides?
Stephen La Rocque and Harley Weston lui répond.
Area of a triangle formed by three points on a graph 2007-10-26
Betty pose la question :
My question is 'find the are of a triangle whose vertices have coordinates (3,5),(6,-5), and (-4,10)
Stephen La Rocque lui répond.
A graph with 100 vertices 2007-10-24
amarjeet pose la question :
let g be a graph with 100 vertices numbered 1 to 100. Two vertices i and j are adjacent only if i-j=8 or i-j=12. The number of connected components in g are:
1. 8
2. 12
3. 4
4. 25

Penny Nom and Victoria West lui répond.
A conical cup 2007-10-18
Nicholas pose la question :
Water is leaking out of a small hole at the tip of a conical paper cup at the rate of 1cm^3/min. The cup has height 8cm and radius 6cm, and is initially full up to the top. Find the rate of change of the height of water in the cup when the cup just begins to leak. Since V= (pi/3)r^2h, how do I eliminate a variable or change the equation so I that I can answer the question? Thanks.
Penny Nom lui répond.
The normal approximation to the binomial 2007-09-30
m.j. pose la question :
Slot Machines The probability of winning on a slot machine is 5%. If a person plays the machine 500 times, find the probability of winning 30 times. Use the normal approximation to the binomial distribution.
Harley Weston lui répond.
The area of a roof 2007-09-27
RANDY pose la question :
I have an area of roof. It's in a sort of triangle. it's 25' across the top 30' at each edge 86' across the bottom
Stephen La Rocque lui répond.
Grade 5 shapes and measurements 2007-09-23
Marina pose la question :
I'm helping my grade 5 son in this assignment but I don't know how. There are 2 question I would like you to help me:
1.How can the shapes used to form a larger similar figure using 4 shapes: ( a 3 by 3square is drawn in a 6 by 6 dot figure,drawing attached)
2.You are planning to build a birdhouse measurement is attached,. You are going to cut the pieces from a board that is 12 inches wide. How long should the board be?

Penny Nom lui répond.
Are the diagonals of a trapezoid ever perpendicular? 2007-09-23
Amanda pose la question :
Are the diagonals of a trapezoid ever perpendicular?
Harley Weston lui répond.
x = y^2 and x = -4 y^2 2007-09-11
Jil pose la question :
My question is when dealing with parabolas, x=y^2, so that they are sifted on their side you could say, what will happen to the graph if you change it to x=-4y^2. I understand that the - flips the graph in the other direction but can you simply just plug in numbers and increase the stretch of the y?
Penny Nom lui répond.
Is there a way I can calculate correct square footage for odd shapes? 2007-08-31
Tom pose la question :
I manage a golf course, and we are about to over seed the course for the upcoming winter months. We will be over seeding all greens, fairways, and tee boxes. Noone of the area that we will be placing seed on is a regular shape. The closest shapes they would be is a circle, rectangle or square, but most of them are all odd CURVED shapes. I know what the equation is for Square ft., and I know how many Square ft. are in an acre, but I need to be precise with the amount of seed I order. Is there a way I can calculate correct square footage for odd shapes.
Harley Weston lui répond.
Applications of sequences and series 2007-08-27
Trish pose la question :
I'm a grade 12 learner working on a math project based on sequences and series. I'd like to know the different types of sequences and series such as fibonacci, fourier, farey, etc.

I've already used the Fibonacci Sequence and Harmonic Series and need two more. The simpler the sequence or series type the better.

I'd also like to know in which non-mathematical areas use sequences and series and how. Areas such as engineering or science.

Penny Nom lui répond.
What feature on a graph shows the solution? 2007-08-19
San pose la question :
To solve the equation 2sinx+square root 3=0, what feature of the graph, in the form y=asinx+b would show the solution? Draw a graph to show the solution. thanks.
Stephen La Rocque lui répond.
Explain how you can shift a parabola up two units 2007-08-14
Marsia pose la question :
Explain how you can shift a parabola up two units (y axis).
Stephen La Rocque lui répond.
Paying off a loan 2007-08-08
Iona pose la question :
I borrowed $6000. 00 from a friend to pay school fee. He said I should not pay back any interest but pay $350.00 monthly after graduation. (1) Make an equation from the above (2) make a graph (3) what is the relationship between the months after graduation and the loan? (4) what is the horizontal and vertical intercept relate to the problem?
Stephen La Rocque lui répond.
The irrationality of PI 2007-08-07
Matthew pose la question :
I have what I like to think of as a rather interesting question that I can't explain confidently for the life of me. If we take a circle with a radius of 1 and we calculate the circumference, we can use 2 pi R. Doing this calculation results in a circumference of 6.28318530717~ which goes on forever. However, if you were to take a that same circle in the real world, say with radius 1cm and wrap a string around it, and then measure the string, you don't get 6.28~, you get something like 6.2, a much more finite distance. The length of the string is not an irrational number, like the math claims it to be.
Claude Tardiff and Stephen La Rocque lui répond.
Subdividing a property into half acre parcels 2007-07-31
Pamela pose la question :
We need to divide a large property with five sides and strange angles into 1/2 acre parcels from the top down. We want to know where along the eastern and western borders of the large property to make the divisions.
Stephen La Rocque lui répond.
The graph of a log function 2007-07-30
Marsia pose la question :
How would you describe to another student what the graph of a logarithmic function looks like?
Penny Nom lui répond.
Dried grapes and percentage of water content 2007-07-09
Jyoti pose la question :
Fresh grapes contain 90% water by weight while dried grapes contain 20 % water by weight. what will be the weight of dry graprs available from 20kg of fresh grapes?
Stephen La Rocque lui répond.
Period of a sum of trig functions 2007-06-17
Aakash pose la question :
the period of the function f(x)=cos3x+sin4x+tan4x
Stephen La Rocque lui répond.
What is the name of the shape that is 3D and irregular? 2007-06-14
alan pose la question :
What is the name of the shape that is 3D and irregular?
Stephen La Rocque and Penny Nom lui répond.
More on quadrilateral shape names 2007-05-26
Don pose la question :
If North Americans call a quadrilateral with no parallel sides a trapezium, is a kite merely a special type of trapezium? Can a rhombus be a kite?
Walter Whiteley and Penny Nom lui répond.
Is a square also a right trapezoid? 2007-05-24
Don pose la question :
Is a square also a right trapezoid?
Stephen La Rocque lui répond.
Finding square roots 2007-05-17
Chandler pose la question :
I would like an easy way to find the square root of a number.
Gabriel Potter lui répond.
Parabolas 2007-05-16
Andy pose la question :
How do you write an equation of a parabola that opens to the left with a vertex of (2,-6)?
Stephen La Rocque lui répond.
Pattern for a truncated cone 2007-05-11
Mike pose la question :
I have been trying to get this cone flat so I can build this column. Can you please help me so I can figure this out? Thanks for your help.
Stephen La Rocque lui répond.
Finding a parabola's equation by looking at its graph 2007-04-30
Kenzie pose la question :
The graph shows an arrow going upward crossing at the -2 on the x line and crossing the 3 on the x line and the vertex on the -6 on the y line.
Stephen La Rocque lui répond.
How do i form a paper cone 2007-04-14
Sash pose la question :
How do i form a paper cone with the height of 25 cm, the slant height of 25.8 cm, and the radius of 6.2 cm?
Stephen La Rocque lui répond.
A sketch of a quadratic equation 2007-04-14
Fee pose la question :
A ball is thrown upward from the roof of a 25m building. The ball reaches a height of 45m above the ground after 2s and hits the ground 5s after being thrown. Use the fact that the realtion between the height of the projectile and time is quadratic to draw an accurate graph of the relation on graph paper.
Penny Nom lui répond.
Graphing a line 2007-03-23
mitchell pose la question :
Graph each equation. y = 2x + 4
Jaymi Peterson and Haley Ess lui répond.
A pictograph 2007-03-14
Angie pose la question :
I am confused! I have to include a title, labels, and a key: Here is the problem

Snickers 45
Kit Kats 65
Nerds 30
Kisses 50
Suckers 75

With the above information please help me make a candy pictograph

I would gladly like your help as my child is a second grader and can not tell me anything about a pictograph

Penny Nom lui répond.
Using complex numbers 2007-03-12
Kara pose la question :
Do you use complex numbers in your job?
Stephen La Rocque and Penny Nom lui répond.
Statistics data 2007-03-08
Parnini pose la question :
The following table shows the expenditure on different household items by a family. Draw a line graph for the following data:
Items : Food Rent Cloth Education Saving Others
Expenditure : 2000 2.500 1.500 1.200 4000 3.500

Stephen La Rocque lui répond.
The area of a decagon 2007-02-28
heather pose la question :
how do i find the apothem and area of a decagon that has a perimeter of 60?
Penny Nom lui répond.
The area of an L-shaped polygon 2007-02-23
Louanne pose la question :
How do I find the surface area of a L shaped polygon?
Melanie Tyrer lui répond.
x^x = 2x 2007-02-22
ramsay pose la question :
This has been bugging me for ages. How do find both real solutions to x^x = 2x? Obviously I have x=2, and there's another at about 0.35, but I can't work it out properly. Any help?
Penny Nom lui répond.
The length of the side of a hexagon 2007-02-19
Nick pose la question :
if the area of a regular hexagon is 259.807 how do i find the length of each side, and the apothem? thanks if you can help
Penny Nom lui répond.
a specific 3-d shape 2007-02-12
Robby pose la question :
What three dimensional shape has 6 vertices, 5 faces, and two congruent triangular bases? I can't find the answer to this question anywhere. Your help will be appreciated. Thanks, Robby Craig
Pam Fowler, Gabe Potter and Walter Whiteley lui répond.
Large exponents 2007-02-09
Nick pose la question :
I am trying to figure out an extremely large number. It relates to the estimated number of bacterial divisions in 12775 generations of bacteria.
The problem I need to solve is:
2^12775 or 2 to the power of 12,775.

Stephen La Rocque and Penny Nom lui répond.
The capacity of a tank 2007-02-08
lorn pose la question :
what is the capacity of a tank height is 110 feet diameter is 24 feet
Brennan Yaremko lui répond.
The cube root of 8000 2007-01-31
Jacqui pose la question :
How would you find the answer of a cube root backwards. For example: 8000 but backwards. Like, what number made that and how would you do it?
Penny Nom lui répond.
Splitting a 5 acre piece of property 2007-01-25
John pose la question :
I have a 5 acre piece of property that I wish to split into 5 individual 1 acre parcels. The 5 acre parcel 1s 660 feet in length and 330 feet in width. The first two parcels are 264'x165', but I am asking for help getting the perimeters of the last 3 lots, one of which is a irregular pentagon. All of the one acre parcels must have an area of 43,560 square feet each.
Penny Nom lui répond.
Line segments on dot paper 2007-01-21
Khaori pose la question :
The three line segments below are drawn on centimeter dot paper.

a. Find the length of each segment to the nearest ten-thousandth of a centimeter.
b. Could these line segments be arranged to form a triangle? If no, explain why or why not. If yes, answer this question: could they form a right triangle? Explain why or why not.

Penny Nom lui répond.
y = x^2 - 2 2007-01-08
Mike pose la question :
Graph the quadratic equation: y=Xsq-2
Penny Nom lui répond.
Are all rectangles trapezoids? 2007-01-05
Sarah pose la question :
Are all rectangles trapezoids?
Chris Fisher lui répond.
A linear system 2007-01-04
Harold pose la question :
System A (4, 2/3)
2x-3y=6 and x-3y=2

I approached this problem using the addition method.

Could i have found the answer using the graphing method?

Penny Nom lui répond.
A trapezoid with one right angle? 2006-12-09
Andre pose la question :
How do you draw a trapezoid with one right angle?
Stephen La Rocque lui répond.
Conic sections 2006-11-19
Joyce pose la question :
My son has a project on conic sections. I need the following information on Parabola, Circle, ellipse,and hyperbola. He can't find the following information for each conic section: equations with explanations, four uses for each shape and Shape explanation.
Penny Nom lui répond.
Motorcycle expansion chamber design 2006-11-14
David pose la question :
I'm interested in calculating cone information regarding motorcycle expansion chamber design for example. I guess it's called a truncated cone, from what I've read so far. If I know the center line height, small radius, and large radius of a truncated cone then, how can I calculate the angle (included angle?) the cone forms? I'd like to know the variations of the formula so I can calculate for angle, or length, or one of the diameters if I know the other two measurements.
Stephen La Rocque lui répond.
Apples and plums 2006-11-13
Saif pose la question :
the problem i am stuck with is, nicole buys 2.3kg apples 1.8kg plums she pays £7.18 total plums cost £2.20per kg cost 1kg apples what is the cost of 1kg of apples
Stephen La Rocque lui répond.
Some applications of conic sections 2006-11-13
Burt pose la question :
how are circles, ellipses, and hyperbolas used in everyday life
Penny Nom lui répond.
The name of a shape 2006-11-11
Valentina pose la question :
I have a shape of 4 sides 3 are the same length (2cm each) and the other smaller (1.5cm)
Chris Fisher lui répond.
y = |x| - 10 2006-11-03
Pam pose la question :
what is the process for finding the vertex point for an equation such as this, y= |x| - 10.
Penny Nom lui répond.
How does knowing the sum for 2+7 help you find the sum for 6+1 2006-10-26
Andrea pose la question :
How does knowing the sum for 2+7 help you find the sum for 6+1
Haley Ess lui répond.
An approximation 2006-10-22
Ellen pose la question :
consider the curve -8x^2 +5xy+y^3 +149 =0 Write an equation for the line tangent to the curve at (4, -1) use this equation to approximate the value of K at the point (4.2, K)
Penny Nom lui répond.
Is a circle a regular shape? 2006-10-10
David pose la question :
Is a circle a regular shape or an irregular shape?
Stephen La Rocque lui répond.
Plotting a linear equation 2006-10-09
Jennifer pose la question :

I am having problems explaining how to do these problems to my child .

graph:2=x-y

graph the equation: 2y+x=-3


Penny Nom lui répond.
The focus of a parabola 2006-10-01
Lily pose la question :
I have a mathematical assignment which includes applications of parabolas, hyperbolas and ellipses in the real world. I have been searching the internet and now I am ware that most of the applications of parabolas have a connection with what people call "the focus". However, I do not think I clearly understand what "the focus" of a parabola is. Would you please explain it to me?
Penny Nom lui répond.
Your birthdays until your one hundredth 2006-09-09
Alan pose la question :
which of your birthdays until your one hundredth, fall on the same day of the week as the day you were born?
Chris Fisher lui répond.
Find the points of intersection 2006-08-18
Ingrid pose la question :

The question says find the points of intersection of the graphs of the equation, and check your results analytically.

x squared + y squared=25
2x+ y=10


Stephen La Rocque lui répond.
A hamiltonian circuit 2006-07-24
Tom pose la question :

1)Prove that every simple not directed graph with 21 vertices and 208 edges has
a hamiltonian circuit but not an Euler circuit.

(I have proved that if we give the 2 edges for each vertices then after a few
steps I have 21 vertices of odd degree and 19 edges.So in the end I have 2
vertices of odd degree.So I have not an Euler circuit.But how can I prove that
there is a Hamiltonian one?)

2)If G is a tree and has a virtex k degree,prove that it also has at least k
vertices degree 1.


Penny Nom lui répond.
Area of a trapezium demonstration 2006-07-18
Karthik pose la question :
how can u " SHOW THAT THE AREA OF THE TRAPEZIUM IS EQUAL TO HALF THE PRODUCT OF ITS ALTITUDE AND THE SUM OF ITS PARALLEL SIDES." in an Innovative And Creative way.....
Stephen La Rocque lui répond.
The area of a sector and a triangle 2006-06-23
Howard pose la question :
I thought of the following problem which is similar but much simpler than the tethered goat problem: What is the angle(it is more illustrative in degrees)of arc of a unit circle so that the area between the chord it subtends and the arc length is equal to the area of the triangle with opposite side the subtended chord.
Stephen La Rocque and Penny Nom lui répond.
As close to 841 as possible 2006-06-21
Alan pose la question :
using these numbers by either add subtract division multiply come up with the answer as close to 841
100 25 10 7 6 3

Paul Betts lui répond.
What will the graph of y=x2 - 2x - 3 be? 2006-06-17
Byrony pose la question :
What will the graph of y=x2 - 2x - 3 be?
Steve La Rocque lui répond.
A truncated cone 2006-05-28
Phil pose la question :
Hi, I am an art student and I am trying to make a "truncated cone" (ie: a cone with the top cut off) out of sheet metal. I need to design a template first and am having problems working out the angles. The truncated cone is 250mm high, 550mm wide at the bottom and 290mm wide at the top. Can you help?
Stephen La Rocque lui répond.
How do you graph 3x + 6y = -9 by axis intercepts? 2006-05-22
Crystal pose la question :
How do you graph 3x + 6y = -9 by axis intercepts?
Penny Nom lui répond.
The interior angles of a right triangle 2006-05-20
Greg pose la question :
I am wondering if there is a way to figure out the interior angles of a right triangle if we know ONLY the side lengths, and the trick is, we CANNOT use arctangent!
Leeanne Boehm and Penny Nom lui répond.
Landscaping 2006-05-11
Luke pose la question :
HOW MANY CUBIC YARDS OF CONCRETE WILL IT TAKE TO FILL UP A BOX 20FT.X20FT.X6IN DEEP?


HOW MANY CUBIC YARDS OF CONCRETE WILL IT TAKE; I HAVE A 12FT.IN DIAMETER POND AND I WANT TO PUT A 3FT.SIDEWALK AROUND THE OUTSIDE OF IT?

Stephen La Rocque lui répond.
Geometric sequence and basic functions (graphs) 2006-04-20
Marlene pose la question :
Which of the basic functions is related to the geometric sequence: Linear, Quadratic, Rational, or Exponential? Can you give me an example of how it would be used in normal life?
Stephen La Rocque lui répond.
Cubic yards of landscape stone 2006-04-19
Janet pose la question :
I need to spread landscape stone around my pool. A landscaper said I need 16 yards. What is the area of a cubic yard if I want the stone 3" deep?
Stephen La Rocque lui répond.
Overlapping area of two circles 2006-04-15
Jade pose la question :
Given two identical circles where the radius (6 units) is the distance between the centers, what is the area of the overlapping region?
Stephen La Rocque lui répond.
The area of a regular hexagon 2006-04-10
A student pose la question :
find the area of a regular hexagon with a apothem of 9
Stephen La Rocque lui répond.
An epicycloid 2006-04-10
Sharon pose la question :
What is the name of the curve formed by a point on the circumference of a circle that rolls on the outside of a fixed circle? This curve is used in the study of gears.
Stephen La Rocque and Penny Nom lui répond.
A table of values 2006-03-28
Cindy pose la question :
Create a table of values for each of the following functions: y=5x, y=52, and y=5x. I dont know to do this.
Stephen La Rocque lui répond.
Linear feet on a paper roll 2006-03-20
Vishal pose la question :
I WOULD WISH TO KNOW OF A STANDARD FORMULA BY WHICH I CAN CALCULATE THE LINEAR FEET OF A PAPER ROLL AS PER THE FOLLOWING DETAILS:-

THICKNESS: 150 GSM (GRAMS PER SQUARE METER)
REEL WIDTH: 85 CM
REEL DIAMETER: 140 CM
REEL INNER CORE: 10 CM
REEL WEIGHT: 946 KG's


Stephen La Rocque lui répond.
The shape of an egg 2006-03-19
Vinson & Julie pose la question :
What is the proper mathematical term for the geometric solid that is represented by an egg?
Chris Fisher lui répond.
A proof by contraposition 2006-03-16
Eban pose la question :

1)by mathematical induction prove that 12 + 32 + 52 + ...... + (2k-1)2 = (1/3)k(2k-1)(2k+1) for all positive integers k.

2)show that the contrapositive of the following statement is true. if 1 + M7 is even, then M is odd.


Stephen La Rocque lui répond.
The area of a parallelogram 2006-03-12
Shayne pose la question :
Is there a method of finding the area of a trapezium or/and parallelogram even if the height is not given? Could you also explain how to find the height?
Penny Nom lui répond.
What names are known for the quarter circle shape? 2006-03-06
Christina pose la question :
What names are known for the quarter circle shape?
Stephen La Rocque and Penny Nom lui répond.
The graph of y = 2x + 1 2006-02-23
Geneva pose la question :
I'm having trouble with graphing equations like y=2x+1 which is my homework I'm not sure how to graph it. I'm in the 8th grade taking a 9th grade course algebra I.
Penny Nom lui répond.
Whose photo is he looking at? 2006-02-19
Adeyeri pose la question :
Looking at a picture a man says, that the father of the person (in photo) is the son of my father. whose photo is he looking at?
Penny Nom lui répond.
Broken-line graphs and histograms 2006-02-16
George pose la question :

1. What is the main difference between a broken-line graph and a histogram? Both represent continuous variables.

2. What is the correct way to read a multiplication array: x-axis first and then y-axis, other way around or it doesn't matter?


Penny Nom lui répond.
Intercepts and slope 2006-01-15
Anthony pose la question :
Given -1.7x + 0.8y=7.1:

(a) state the intercepts and slope for the graph

(b) Draw the graph

Penny Nom lui répond.
Folding a sheet of paper 2005-12-15
Victoria pose la question :
The current problem is to take a normal 8 1/2 x 11 sheet of paper, take a corner and fold it to meet the opposite corner, and (without actually measuring) produce a formula to describe the result fold/crease.
Penny Nom lui répond.
Four tangent circles 2005-12-06
Ananth pose la question :

I have one bigger circle A with radius 15.

Inside this bigger circle i have another circle B with radius 3 which touch this bigger circle. Have another circle C with radius 4 which touches A and B. I would like to draw a biggest circle which touches A,B and C.


Chris Fisher lui répond.
Graphing two lines 2005-11-20
A student pose la question :
About y = 5x + 24 and y = -3x - 8
1. Graph both equations on the same set of axes. When you made your graph, how did you choose the range of x and y values?


2.Find the point of intersection for the graphs.


3. Test the point of intersection you found by substituting its coordinates into the equations. Do the coordinates fit the equations exactly?

Penny Nom lui répond.
What shape do I say a football is? 2005-11-14
Peter pose la question :
I am a 43 year old man. I am no longer in school, I have a bachelors degree. I like to tell stories about sports in ways to make people think about the amount of time they devote to the sports. For example, while attending a church in Chicago on the north side, I would pass by Wrigley field. I began to call it the church of the flying orb or sphere. Now if I want to tell a similar story about football (not soccer), what shape do I say a football is?
Chris Fisher lui répond.
Shapes 2005-11-12
Tonya and Hailey pose la question :
MY THIRD GRADERS TEACHER HAS SENT HOME HOMEWORK ASKING FOR THEM TO FIND REAL-WORLD OBJECT IN MANY DIFFERENTS SHAPES A FEW HAVE US STUMPED
Penny Nom lui répond.
The diagonals of a trapezoid 2005-11-11
Chris pose la question :
Is there ever a case where the diagonals of a trapezoid bisect each other?
Walter Whiteley lui répond.
Painter's caps 2005-10-19
Duk pose la question :
The equation C=10 +2n represents the cost in dollars, C, for n painter's caps advertising the walkathon. Which pair of values could represent a number of caps and the cost for that number of caps, (n, C)?

(0, 10) (7, 24) (15, 30)

Penny Nom lui répond.
Open dots and closed dots 2005-09-29
Cynthia pose la question :
When graphing the solutions of an inequality, what is the difference between an open dot and a closed dot?
Penny lui répond.
A 3-dimensional pie shape 2005-09-17
Bill pose la question :
Your site appeared in my search for the name of a 3-dimensional pie shape. 2-d would be a sector of a circle. As it it curved, I don't believe it is in a the polyhedra family. Can you help me find the mathematical term for it?
Chris Fisher lui répond.
Graphing a linear inequality 2005-08-26
Gina pose la question :
When graphing a linear inequality, how do you know if the inequality represents the area above the line?
Penny Nom lui répond.
Graphing an inequality 2005-05-02
Janice pose la question :
I need to know how to graph inequalities. I need to know just abot everything. From graphing an equation to shading.
Leeanne Boehm lui répond.
Area of a region on a map 2005-04-28
Balachandar pose la question :
Pl. find the attached map.

1. Can u pl. find out the total square are of WLMZ
2. Can u pl .find out the total square are of WXYZ -- { A }
3. Can u pl .find out the total square are of XLMY -- { B }

Pl .help me to find out the above.

.....


Chris Fisher and Penny Nom lui répond.
A roll of paper 2005-04-16
Tuomas pose la question :
How would you calculate a diameter of a paper reel when the paper is 0,04 millimeters thick and for example 5000 meters of it is winded on a 76 mm (3inch) core.
Penny Nom lui répond.
Two overlapping circles 2005-03-20
Safi pose la question :
I have a problem to calculate the area of two overlapping circles because two circles are overlap then how i calculate the overlap area to subtract from the area of both circle.
Penny Nom lui répond.
The capacity of a dryer 2005-02-23
Darlene pose la question :
I am trying to figure out the capacity of my dryer. I need to know the size in order to buy a new one. The drum within measures 27" in diameter and 18" deep. I know that I am looking for an answer in cubic feet but am not sure how to determine this.
Penny Nom lui répond.
Graphing inequalities 2005-02-15
Melanie pose la question :
Solve and Graph the following variable inequalities:

y<2x+4

y>-2x+6

3x+4y<-24

4x-6y>-24

Penny lui répond.
The diaonals of an isosceles trapezoid are congruent 2005-02-02
Daniel pose la question :
I am currently unable to find a proof that the diaonals of an isosceles trapezoid are congruent. Might you happen to have one?
Walter Whiteley lui répond.
Approximating an integral 2005-01-27
A student pose la question :
i used excel to find the area under a graph instead of integrating the function the analytical way. These are the different answers i got with different width of the x-values, the smaller the x-increment gets the more precise the area is when compared to the answer obtained the numerical way. I have trouble seeing why the answers i get is getting bigger and bigger and not smaller can someone plz explain?
Penny Nom lui répond.
The area of a trapezoid 2005-01-15
Doak pose la question :
What is the total acreage of a pie-shaped piece of land that is:
146' f
ront 85' back
246' each side

The front and back lines are fairly straight....a trapezoidal shape.

Penny Nom lui répond.
An isosceles triangle 2005-01-03
Abraham pose la question :
The question is,"Triangle ABC is not isosceles.Prove that if altitude BD were drawn, it would not bisect AC."My question is If an altitude is drawn wouldn\'t that mean automatically its isosceles because, In a triangle the sides opposite congruent angles(in this case the right angles)are congruent? What am I thinking wrong?
Harley Weston lui répond.
A graph with certain properties 2004-11-22
A student pose la question :
i was asked as a question in coursework to sketch the graph with the following characteristics:
a double root at -3
a pair of imaginary roots
an x-intercept at 6
a root at 4 which is not a double root

Penny Nom lui répond.
The points of intersection of two graphs 2004-11-05
Benjamin pose la question :
How do I find the points of intersection of the two functions:
1) y =
2 - e-x 2) y = 1 + x2

Harley Weston lui répond.
A theorem involving a trapezoid 2004-09-29
Abraham pose la question :
Given:Trapezoid ROSE with diagonals RS and EO intersecting at point M
Prove:Diagonals RS and EO do not bisect each other.

Harley Weston lui répond.
The area of a lot 2004-09-28
Stan pose la question :
I own a parcel of land (lot 20) that is assessed at 9127.00 square feet and has a frontage of 99.90 feet according to the tax roll. I don't know how large my property actually is since I feel that the tax assessment and the survey map don't jive
Harley Weston lui répond.
f(x) = x-4 if x<2 2004-09-21
Dani pose la question :
Hi, my name is Dani. I'm in tenth grade Algebra 2, and I'm learning about a lot of different kinds of functions, and I can't quite seem to understand how every part of the piecewise function works. I understand most of it, I think, but I've been looking at an example in my math book, and I'm not sure how they got the answer they did. The part I don't understand reads: "Graph f(x) = x-4 if x<2" and "Identify the domain and range." The picture in the book shows an open circle at (2,-2), with an arrow going down and to the left with the slope being one. How did they get the open circle to be at (2,-2), and why does the arrow point where it does?
Penny Nom lui répond.
Constructing a cone 2004-09-07
Steve pose la question :
I am trying to build crayfish traps; one of the components is a cone shaped entry section.

The cone I want to make would be 12" in diameter at the base and 12" in height, from base to peak.

I need a formula to calculate the dimensions and a method of transferring the shape onto a flat piece of material.
Harley Weston lui répond.

An isosceles trapezoid 2004-08-31
Bruce pose la question :
An isosceles trapezoid with bases of lengths 12 and 16 is inscribed in a circle with a radius of 10. The center of the circle lies in the interior of the trapezoid. Find the area of the trapezoid.
Penny Nom lui répond.
The intersection of two graphs 2004-07-28
JJ pose la question :
Is there a way to find the intersections of these graphs algebraically?

x^2 + y = 4 & 2x - y = 1

I got (1.45, 1.9) and (-3.45, -7.9) with a graphing calculator.

AND THESE...

y = 3.29x & y = 5.5(x^0.5)+ 10000

I got x at 3133 with a graphing calculator.

Penny Nom lui répond.
The commuter 2004-07-14
JJ pose la question :
A student who commutes 27 mi. to college remembers, after driving a few minutes, he forgot a term paper. Driving faster than usual, he returns home, gets the paper, and once again starts to school. Sketch the graph of the student's distance from home as a function of time.

Does it mean that the student drives at normal speed, drives faster to pick up the paper, and resumes normal speed to head back, or that the student drives at normal speed, then drives faster the rest of the way to college?

Penny Nom lui répond.
12(1.221)^t = t + 3 2004-07-14
Jay pose la question :
This problem has me stumped since i cannot separate the t and the 3. Here it is:

12(1.221)t = t + 3

It's on a logarithm worksheet, but how do i figure it out?!

Harley Weston lui répond.
Programming without trig functions 2004-05-25
Derek pose la question :
I am a programmer trying to calculate the following.

What is the formula to find the cross-sectional area of a cylinder with out using any trig functions? or better yet, how can you calculate any given volume in a cylindrical tank with spherical heads with out trig functions?

I am using a PLC (programmable logic controller) to do this and trig functions are not available.

Harley Weston lui répond.
The problem of Apollonius 2004-04-25
Mitja pose la question :
There are given 2 circles lying one out of another and one point out of both circles. How to construct a circle passing through a given point and internally tangent to one and externally tangent to the other cirlce?
Chris Fisher lui répond.
The intersection of two graphs 2004-03-31
Benjanim pose la question :
How do I find the points of intersection of the two functions:

1) y = 2 - (e^-x)
2) y = 1 + (x^2)

I know that I have to set them equal to each other, but I can't solve for x. Please help :-)

Penny Nom lui répond.
The degree sequence of a graph 2004-03-31
Ali pose la question :
How can i find out {6,6,5,5,5,3,2}is not the sequence of the points of a simple graph,without drawing it?
Penny Nom lui répond.
A fifteen sided polygon 2004-03-05
Joann pose la question :
WHAT IS THE NAME OF A FIFTEEN SIDED SHAPE?
Penny Nom lui répond.
Napoleon's theorem 2004-02-27
David pose la question :
How do i prove this : For any triangle, if you make 3 equillateral triangles using the sides of the the original triangle, the central points of the 3 tringles another triangle that is equillateral.z
Chris Fisher and Penny Nom lui répond.
Nonconvex shapes 2004-02-18
Merrill pose la question :
Draw 4, 6 and 8 sided shapes so that ONE straight line can be drawn through each side
Chris Fisher and Penny Nom lui répond.
A pyramid-shaped tank 2004-02-13
Annette pose la question :
The base of a pyramid-shaped tank is a square with sides of length 9 feet, and the vertex of the pyramid is 12 feet above the base. The tank is filled to a depth of 4 feet, and water is flowing into the tank at a rate of 3 cubic feet per second. Find the rate of change of the depth of water in the tank. (Hint: the volume of a pyramid is V = 1/3 B h , where B is the base area and h is the height of the pyramid.)
Harley Weston lui répond.
Graphing lines 2004-02-08
A student pose la question :
how do you solve y=1\3x+3 and y=1\3x-3 by graphing?
Penny Nom lui répond.
Logarithms 2004-02-01
A student pose la question :
Where did the word logarithmic come from? Where/whom did it originate?
Harley Weston lui répond.
The trapezoidal rule 2003-12-31
Prashanth pose la question :
I have a set of XY co-ordinates(say 49 points),I want to find the area enclosed by the curve joining them and the axes.I do not have an equation for the curve and I am not able to fit a curve given the points.I use the trapezoidal rule (Y1+Y2/2)*(X2-X1), Is it right? can I do this with any other method?
Penny Nom lui répond.
Intercepts 2003-11-04
A student pose la question :
I am totally confused on how to determine intercepts of an equation and then to graph the line. For example, how to determine the intercepts of 2x-3y-12=0.
Penny Nom lui répond.
Trees 2003-10-28
Ali pose la question :
Is there any formula for counting the number of trees (graphs)?
Denis Hanson lui répond.
A parabola 2003-10-24
Delores pose la question :
Given the vertex (4, -2) y intercept = -6 find if/where the parabola crosses the x axis?
Penny Nom lui répond.
The sketch of a graph 2003-10-07
A student pose la question :
I was wondering how do you figure out if a graph has a horizontal tangent line. One of my homework problem was to sketch the graph of the following function; (4/3)x3-2x2+x. I set f''(x) ( the second derivative) of the function equal to zero and got the inflection point:(1/2,1/6). Also i am having trouble finding the concavity for x>1/2 and x<1/2, i am getting a different answer from the back of the book, the graph i draw looks completely different from the correct answer.
Penny Nom lui répond.
Functions, graphs and derivatives 2003-10-05
Jathiyah pose la question :
I wanted to know how would you tell (on a graph diplaying two funtions), which funtion is the derivative of the other?
Walter Whiteley lui répond.
Applications of logarithms 2003-09-16
A student pose la question :
I have a project due in my algebra two class dealing with logs and where they are used in life, but I am having trouble finding websites that relate. So I would really appreciate it if you couldhelp me.
Penny Nom lui répond.
Graphing a piecewise function 2003-08-24
Amber pose la question :
How do i begin to graph a piecewise function, absolute function or step function?
Penny Nom lui répond.
The area of an odd shaped piece of property 2003-07-24
Barry pose la question :
What is the formula for figuring the square footage of an odd shaped piece of property?
Penny Nom lui répond.
Some shapes 2003-04-29
Suzanne pose la question :
  1. What shape has 12 edges all the same length?

  2. What shape has 3 surfaces and 1 is curved?

Penny Nom lui répond.
Two intersecting graphs 2003-04-23
Patty pose la question :

a) graph the equation

x2 - y - 4 = 0

x2 + y2 = 9
on the same set of coordinate axes.

I did not have a problem with this.

The problem is part (b) of the question ask: Find all solutions of the system in part (a) algebraically. Express answers in decimal form, accurate to two decimal places.


Penny Nom lui répond.
Area of a trapezoid 2003-04-13
A road builder pose la question :
My husband works with asphalt building roads. There are times when one end of the road will be (for example) 100ft wide the other end would be( for example) 200ft wide and he must figure the area in square feet. So far it has been a guessing game because he dosen't have the formula to figure the square feet.
Penny Nom lui répond.
y < 2x + 1 2003-03-01
Erika pose la question :
how can I solve and graph y<2x+1 and which side in the graph should i shade after i finish solving my inequality?
Penny Nom lui répond.
y = 1 - sin(x + 60) 2002-12-10
Eman pose la question :
Sketch the graph of y = 1 - sin(x+60). for 0 <= x<= 360, giving the coordinates of the maximum and minimum points and the pints where the curves crosses the y axis.
Penny Nom lui répond.
A lampshade from a cone 2002-11-26
Ellsie pose la question :
I need to make a pattern to cover an old lampshade. This is actually the bottom portion of a cone. Please help me figure out how to draw this pattern, so that we can complete our project.
Penny Nom lui répond.
Performance reviews 2002-11-21
Lara pose la question :
Perhaps you can help.

He's doing performance reviews at work.

The average performance review is 3.92 on a 5.0 scale. This person would receive a 3.5% raise for next year.

The highest performance review is 4.9 and this person would recieve a 6.0% raise for next year.

How do I solve for the other performance review numbers.


Penny Nom lui répond.
Rounding 27.27 2002-11-03
A parent pose la question :
If you have a problem 27.27 and you need to round to the first 7 the answer would be 27

Would 27.0 be acceptable as well or is it completely wrong.


Penny Nom lui répond.
The four colour theorem 2002-10-27
Rhonda pose la question :
is there an actual equation for the four colour map theorem??? i cannot find it anywhere!! if so can you give a breif description on how it works!!
Penny Nom lui répond.
x 4 + x 5 = 100 2002-10-27
Bill pose la question :
One of my students has stumped me. He asked how to solve the equation 4 x + 5 x = 100

All I can think of are graphing methods to get an approximate solution. What am I missing?


Harley Weston lui répond.
Intercepts 2002-09-04
Tamara pose la question :
Why is it that the x intercept the value of y is 0 and at the y intercept the value of x is 0?
Leeanne Boehm lui répond.
A schedule for a fantasy football league 2002-09-01
Steve pose la question :
i am in a ten(10) team league, split evenly into two five team divisions. we play a thirteen(13) game schedule and would like to play each team in your own division twice, and each team in the other division once. according to my rudimentary mathematics that equals 13 games. a good schedule should look somthing like this: team 1-5 = division 1 team 6-10 = division 2 team 1 should play teams 2-5 twice and teams 6-10 once.
Claude Tardif lui répond.
When does an inequality represent the area above the line? 2002-08-25
Tamara pose la question :
How do you determine that an inequality represents the area above the line and when do you include the line in the solution?
Penny Nom lui répond.
Conics 2002-05-29
Brooke pose la question :
Which conic cannot be generated by an intersection of a plane and a double napped cone?
Chris Fisher lui répond.
Overlapping circles 2002-05-29
Naman pose la question :
There are two circles, big circle with radius R and small one with radius r. They intersect and overlap in such a way that the common area formed is 1/2 pi r 2 (half the area of the small circle) If r=1, find the Radius of the big circle (R)?
Harley Weston lui répond.
A polynomial 2002-04-14
A student pose la question :
A polynomial function is described by the following conditions: f(x) has two real zeros at x= -2 and x= 1, each of multiplicity 2 f(x) has two complex zeros as x goes to -infinity, f(x) goes to +infinity as x goes to +infinity, f(x) goes to +infinity f(x) has a y-intercept at (0,2)

Sketch a polynomial function that satisfies the above conditions.

Write an equation for function f(x)

Write another function g(x) that also satisfies the above parameters.


Penny Nom lui répond.
Cube roots 2002-03-26
Kathy pose la question :
My son is in 4th grade and needs to simplify the cube roots of 216, 512 and 64. How does he do this with a regular calculator or without?
Penny Nom lui répond.
x and y-intercepts 2002-03-10
A student pose la question :
I have the problem- f(x)= X-5/X2+X-6 and I have to find the vertical asymtope, horizontal asymtope, x-intercept, and y-intercept. And graph the problem. I am having problems finding the y-intercept.
Harley Weston lui répond.
When will the calendar be the same as 2002? 2002-02-20
John pose la question :
When will the calendar be the same as 2002? I know the answer is 2013, but I don't know how to answer this mathematically, or if it is possible to do so. This is a ninth grade level algebra question my daughter received from her teacher. Any ideas?
Chris Fisher, Patrick Maidorn and Penny Nom lui répond.
Papy's Computer 2002-01-11
Mary-Anne pose la question :
My second-grade son brought home a fun math worksheet which involved calculating sums using "Papy's Computer." I had never heard of this before and found it to be quite interesting. Each digit in a decimal number is represented by a 2x2 grid. Each grid square corresponds to one of the numbers 1,2,4, or 8.
Harley Weston lui répond.
Maple trees and pine trees 2002-01-06
A student pose la question :
A team of scientists found that in a forest there was four maple trees for every nine pine trees.How many maple were there if they counted 45 more pine than maples?
Penny Nom lui répond.
A quadrilateral with 0 sets of parallel sides 2001-12-12
Jess pose la question :
Ok well for quadrilaterals a parallelogram is a 4 sided figure with 2 sets of parallel sides and a trapezoid is a 4 sided figure with 1 set of parallel sides.

So is there a name for a quadrilateral with 0 sets of parallel side?


Chris Fisher lui répond.
Applied geometry 2001-11-02
Jenny pose la question :
Where can I find some hands-on activities for my Applied Geometry classes? I want to do more activities with them that allow us to get out of the classroom. However, I want to use activities that use only inexpensive equipment because I usually buy the equipment myself.
Walter Whiteley lui répond.
The median of a trapezoid 2001-10-01
Laura pose la question :
Given: A (1,2) B (9,-2) C (7,2) D (3,4)

Find the endpoints of the median. Use exact values. Write the equation using the letters from the given trapezoid. Verify the theorem using algebra.


Penny Nom lui répond.
An egg shaped island 2001-09-22
Karen pose la question :
I am a civil engineering designer trying to design an egg shaped island. I want a 30' radius at the top and a 40' radius at the bottom and the longest length of the egg to be 125'. Is there standard geometry for an egg shape? I am not held to exact radii or the length given.
Chris Fisher lui répond.
Double bar graphs 2001-09-09
Leslie pose la question :
My daughter has a homework problem in which she is graphing data from two frequency tables onto a double bar graph. The graph shows how many students worked in a program in two different years, grouped by age. In the first age group, one of the years has no students working, while there were students in that age group in the other year.
Penny Nom lui répond.
Graphing equations 2001-05-01
A student pose la question :
If, for example, 4x2-12x+9=0 and -4x2+12x-9=0, which I'm assuming it does since you can derive that equation from the first, why do those two equations have different graphs?
Penny Nom lui répond.
f(x), f(x) + 2, f(x +2) 2001-04-04
Monica pose la question :
Could you explain to me how one should go about graphing functions such as f(x), f(x+2), and so on. Also, how should you explain things such as constants and relationships among functions?
Penny Nom lui répond.
Approximating 260 divided by 3 2001-04-02
Andrew pose la question :
Choose the letter of the best estimate.

260 divided by 3

a. 60
b. 70
c. 90
d. 100


Paul Betts lui répond.
Dots on squared paper 2001-03-30
Tom pose la question :
Consider some squared paper with lines 1cm. apart and dots where the lines cross. Place squares whose sides are whole numbers of centimetres on the paper in such a way that the sides of the squares always lie along the lines on the paper. For each square, find (a)the number of points inside the squares; (b)the number of points on the square and (c)the area of the square. How are these quantities related?
Leeanne Boehm and Penny Nom lui répond.
Cube roots on a calculator 2001-03-24
Will pose la question :
Hi my name is Will.I have a question about the calculator TI-83 Plus or the use of a scientific calculator. It is about using them to turn a cubed root to a decimal. When using that for a rational and irrational number. Rational numbers is a number that terminates or repeates. A irrational number goes on and on and uses ... (like pie). Like when you type the cube root of 8 it gives you 2, and that is a rational number. The squre root of 2 is 1.41421356... that is irrational. So why is it when you put the cube root of 16 in the calculator it says 2.5198421 that is rational it terminates at the ninth digit and my float is set for the tenth? But why when you do it by hand you get the 2 times the cube root of 2 and that is irrational? Why is that?
Judi McDonald lui répond.
Shape names 2001-02-26
Paul pose la question :
I have been asked to give names of shapes from A to Z . I can only find 14 shapes can you help..... I'm missing letters b, g, j, l, m, n, u, v, w, x, y, z.
Harley Weston lui répond.
Shape, symmetry, measurement and space 2001-02-24
Marilyn pose la question :
I have to write in detail about how young children learn about shape, symmetry, measurement and space. It is not activities I need it is how the children learn. How/why their brain takes it all in
Walter Whiteley lui répond.
What is my name? 2001-02-23
Julia pose la question :
My perimeter is 12 cm. My sides are all equal. The sum of half my sides is equal to 6 cm. How many sides do I have? What is my name?
Penny Nom lui répond.
Finding roots 2001-02-01
A student pose la question :
My math problem is right now we are working on roots. I don't quite understand how to find the answer to the problems, i was wondering what is the easiest, and fastest way to find the answers to roots?
Penny Nom and Claude Tardif lui répond.
Polynomials and exponents 2001-01-15
A student pose la question :
I am duing a project in math on polynomials and exponents. I need a real life usage of polynomials and exponents for my project.
Penny Nom lui répond.
The Pythagorean Theorem 2001-01-08
Megan pose la question :
Why the Pythagorean Theorem so important in our lives and what is it's history?
Penny Nom lui répond.
The pythagorean theorem in everyday life 2001-01-06
Josh pose la question :
What are some ways that we use the pythagorean theorem in jobs, or even in everyday life?
Claude tardif lui répond.
Solving x - sin(x) = constant 2000-12-29
Keith Roble pose la question :
If x is in radians, how do you solve for x, where: x-sin(x) = constant?
Harley Weston lui répond.
A Trapezium problem 2000-12-15
Ben pose la question :
A trapezium has a perimeter of 22cm. The 2 parallel sides are such that the length of one is three times the length of the other. The non-parallel sides are equal. If the distance between the parallel sides is 4cm, find the lengths of the 4 sides.
Claude Tardif lui répond.
Parabolas in life 2000-12-03
Ashley pose la question :
I am a student and my teacher recently gave us the assignment of writing a portfolio on parabolas in life and finding examples, three to be in fact, only we have to go into detail about only one. We have been instructed to include such terms as: axis of symmetry, completing the square, parabola, quadratic formula, standard form (vertex form) and vertex. We also must include in our detailed example an equation of the parabola and very specific details, PLEASE HELP!
Harley Weston lui répond.
Graphing F(x) = 3x 2000-11-06
Jose pose la question :
graph the exponential problem F(x)=3x
Harley Weston lui répond.
Tapes and CD's 2000-10-22
Arnold pose la question :
Nicole has 25 CDs and tapes altogether. When she tries to pair them up, she has 4 CDs left over. How many tapes does Nicole have?
Penny Nom lui répond.
The scale on a map 2000-10-21
Caroline pose la question :
On a map scale of 1:15000 the distance between two buildings is 31.4cm what is the actual distance in km.
Penny Nom lui répond.
Where will we use this in the real world? 2000-10-11
Jane Ann Musgrove pose la question :
As a teacher of mathematics, I am always asked "Where will we use this in the real world?". I am seeking ideas/sites via the internet where students can find answers to this type of question. Can you help me?

To be more specific, right now I am interested in finding careers where the employees would use the concepts of "Radicals", "Matrices", and "Logarithms". This information will be used by students to make presentations to the class on their findings from internet searches.


Harley Weston lui répond.
Leap Year 2000-09-20
Jennifer pose la question :
The earth revolves around the sun in 365.24 days. How does our calendar deals with the extra 0.24 day?
Penny Nom lui répond.
Graphing an inequality 2000-08-02
Lori pose la question :
How do I find x and y and graph this problem

3x + y < 5


Penny Nom lui répond.
Making a paper cone 2000-07-30
John pose la question :
The question of how to lay out & cut out of paper, cones came up. I would like the cone have :
A base of 4 inches
and height of
4 inches,
6 inches,
8 inches.

Harley Weston lui répond.
An Area 2000-06-26
Frank pose la question :
Can you tell me what the sq. ft. are of and area measuring:
            __________58____________
           |                       | 
            |                      |  
             |                     |    
              |                    |    
               |                   |   
             76 |            70    |       
                 |                 |        
                  |                |        
                  |_____21________ | 

Harley Weston lui répond.
4 Color map Theorem 2000-06-02
Rick Slen pose la question :
I am challenging my students to find a map that can not be coloured using only 4 colours. I know that it is not possible, but one boy drew a map with a "country" totally surrounding all the others so of course it touched all other colours. How do I explain that this type of map is not permissable?
Denis Hanson lui répond.
Graphing a linear function 2000-05-17
Chelsea pose la question :
I need help with grahing linear functions.If you could e-mail me back the basics and how tos I would be much appriciative.
Penny Nom lui répond.
A matrix equation 2000-05-14
A student pose la question :
Right now, we are dealing with matrices and we are supposed to solve the following problem on our graphing-calculators: 2a+3b-4c+d=20
a-2b+3c-5d=-14
3a+4b-2c+3d=19
5a-b+6c+4d=-5

Penny Nom lui répond.
Two calculus problems 2000-05-01
Kaushal Shah pose la question :
How Do WE Integrate the following Functions,
  1. Integral xtanx dx
  2. How was natural base "e" discovered and why e=2.7.......

Claude Tardif lui répond.
Graphing cube roots 2000-04-27
Heather Jones pose la question :
What do the graphs of cube roots look like?
Walter Whiteley lui répond.
Rounding 2000-04-15
Tawana Brown pose la question :
I was uncertain as to whether a number such as 85.25 rounding to the nearest tenth would be 85.3 or 85.2. I thought I had heard somewhere that if 5 is the last number that you don't round up. But I have also heard that anything 5 and above you do round up. It's been awhile since I've done rounding, so I need a refresher course.
Walter Whiteley and Harley Weston lui répond.
Estimates 2000-04-11
Sue Jordan pose la question :
I am looking for examples of estimates for underestimating and overestimating. When would you do each one. For my child age 11 6th grade.
Penny Nom lui répond.
Putting in a pool 2000-03-16
Katie pose la question :
If your digging a hole for a pool and the pool is:

Length= 11m
Width= 6m
Sallow Depth= 1.1m graduating to
'Deep' Depth= 1.8m

What is the volume of soil that will be taken out? And if a bobcat can excavate and remove 10m3 (qubed) of soil an hour how long will it take him to excavate the soil.


Penny Nom lui répond.
A valentines project 2000-02-21
Courtney Dohoney pose la question :
I was assigned a valentines project. Our job is to make a valentines card that has a math theme. On my valentines card I would like to include the set of equations that when graphed look like a heart. If you can figure out a set of equations and get it back to me I would be very happy.
Harley Weston lui répond.
Functions 2000-01-23
Tara pose la question :
Hi my name is Tara, I have two math problems that I need help with in my calculus math class.

  1. If f(x)= x - 2 show that (x+3)f(x)-(x+2)f(x+1)+4=0

  2. Graph this function and use the graph to determine the range y=2x2 - 8x - 3

Harley Weston lui répond.
A roll of paper 2000-01-15
Richard pose la question :
I have a roll of paper, wrapped around a corrugate core, whos diameter is 10.750 in. The outer diameter of the roll is approx. 60 in. The thickness of the paper is .014 in. I am trying to find out how much linear feet of paper is left on the roll, given only the diameter of paper remaining on the core.
Chris Fisher and Harley Weston lui répond.
Bar graphs and histograms 2000-01-11
Raeluck pose la question :
What is the difference between a bar graph and a histogram?
Harley Weston lui répond.
Factoring by graphing 2000-01-11
Dylan Kirkwood pose la question :
How can you factor a quadratic equation by graphing it?
Penny Nom lui répond.
Capacitor discharge rate 2000-01-09
Bill Phillips pose la question :
I need to be solve for t in the following rate problem for an electrical capacitor. Vr = E(e^-t/RC), e=nat. log 2.718 raised to the -t/RC power.
Harley Weston lui répond.
Y-intercept 2000-01-03
Kayla Boylan pose la question :
How do you use and find the y-intercept?
Penny Nom lui répond.
Graph of a sine function 1999-12-23
Pierre pose la question :
Given;
amplitute:1
period: 540
Phase shift: 60 degree,right

I am ask to right the equation: sin 2/3 (value -60degree)

When I am asked to graph the equation, the period is mixing me up.
Harley Weston lui répond.

Approximations 1999-12-20
Adrian Valc pose la question :
Long-time ago I red an article about surprizing (and in a way frustrating) results in math, for example the limit of a nicely defined infinite series which was believed to be a simple rational number, but later was determined to be a transcendent number that missed the rational value by an incredibly small amount (so for example the limit proved to be 2.75000...00137.. with a lot of 0's in between). I cannot find that article anymore, so i was wondering if you have any such examples, or you can point to any relevant information source?
Chris Fisher lui répond.
Cutting a carpet 1999-12-15
Heather pose la question :
A rectanglular piece of carpeting is 90 inches long and 90 inches wide. How can the carpet be cut into two pieces of equal sides and shape to cover an area of 100inches? There can only be one cut and no scraps. Please show me how.
Penny Nom lui répond.
Two calculus problems 1999-12-13
Alan pose la question :
I have 2 questions that are very new to me, they were included on a quiz and the material was never covered. Our teacher never explained the purpose and detailed explanation of how to solve the problem. Could you help? Thanks.

Question 1:
A ball is falling 30 feet from a light that is 50 feet high. After 1 sec. How fast is the shadow of the ball moving towards the light post. Note that a ball moves according to the formula S=16t^2

Question 2:
How many trapezoids must one use in order for the error to be less than 10^-8 if we want to find the area under the curve Y=1/X from 1 to 2. Find the exact area, Graph the function and use the trap rule for the "N" that you found.


Harley Weston lui répond.

Age in minutes 1999-11-21
Mesha Jackson pose la question :
How old would I be in minutes if I were 12
Penny Nom lui répond.
Crossing number 1999-11-06
Christian pose la question :
The crossing number of a graph G, denoted cr(G) is defined to be the minimum number of (pairwise) crossings of edges among all drawings of the graph in the plane. For example, cr(K5)=1 and cr(K3,3)=1.
What is cr(K7,7)?

I figured out that the answer is 81.

Now I am trying to figure out if K7,7 can be drawn in the plane with less than 81 crossings?

I'm not sure how to approach this one. Other than actually drawing it out and checking by trial and error, I am not sure how to approach this problem. Please help!
Denis Hanson lui répond.

A pictograph 1999-10-28
Ali pose la question :
Please tell me how to make a pictograph.I am 2 grade student.
Penny Nom lui répond.
Calculating a square root 1999-10-28
Jonathan pose la question :
What is the formula for calculating a square root?
Penny Nom lui répond.
Broken line graphs 1999-10-19
Lori Samara, Kristi Cameron and the 4th grade students pose la question :
Students in Ms. Cameron's grade 4 class were wondering if a broken line can be used anywhere in the range on the y - axis when creating a broken line graph? They understand that the broken line can be used at the 0 on the broken line but what if there is a large gap in the data from 100 - 200? What do we do? The text book and mathematics dictionary do not answer these questions.
Penny Nom lui répond.
Regular and irregular shapes 1999-10-03
Samuel Tighe pose la question :
What is the difference between a regular shape and an irregular shape? Are a rectangle and a triangle regular or irregular shapes?
Walter Whiteley lui répond.
Intercepts 1999-09-24
Cassandra pose la question :
My book says to find where the X and Y intercept, i dont understand who to do this problem. Can you please help here is the problem. it didnt quite explain the instructions.
Penny Nom lui répond.
Square roots without a calculator 1999-09-14
Josh Weiner pose la question :
Is there any way to find out a square root without a calculator?
Harley Weston lui répond.
Stem and Leaf Plot 1999-09-14
Jeanette Sovick pose la question :
My 5th grade son brought home a math paper, the title of which reads, Reading Stem-and-Leaf Plots...can you explain this so I can explain it to him...There is no book, his teacher just sent this practice sheet home for him to complete and I have no clue!
Penny Nom lui répond.
Spreading a rumor 1999-09-13
Bornstar14 pose la question :
There is someone who wants to spread a rumor within 7 days to 250,000,000 people. he wants to know how many people he should tell on the first day (assuming the rumor is passed by everyone who was told to the same amount of people) so that everyone knows on the 7th day. What I did is the 7th root of 250,000,000. this answer is not possible unless rounded. is it okay for the number to be approximate?
Penny Nom lui répond.
Liquid capacity 1999-06-15
Bernard Antill pose la question :
Would you please tell me how many litres of water in a swimming pool that is 24 feet long, 16 feet wide & 4 feet deep. I would appreciate you showing the formula for this calculation. I am a 74 year old male and I cannot find a formula in which I have any faith! I would also appreciate the answer in Imperial gallons.
Penny Nom lui répond.
Graphing a linear equation 1999-05-27
Dylan Bradley pose la question :
Im in Grade ten and am doing linear equations, I cann't figure out how to make a chart for questions like y = -2x + 3
Penny Nom lui répond.
Shape-preserving transformations 1999-05-04
J McAndrew pose la question :
A shape preserves its shape if a rotation, translation or scaling is performed on it. Are these the only continuous transformations which have this property? These transformations if performed on the parts and then summed have the same effect as the transformation being applied to the whole; are these linear transformations? Who, and what area of mathematics has classified all transformations of this type completely?
Chris Fisher lui répond.
Parabolic shapes 1999-05-04
Justin Ailor pose la question :
Can you give me some parabolic shapes?
Penny Nom lui répond.
Dotted graph paper 1999-04-08
Bridget Winward pose la question :
A teacher at our school is trying to locate dotted graph paper online or in print. His class would like to make three dimensional, geometerical drawings.
Please let us know if you have a good source.

Jack LeSage lui répond.
Universal Area 1999-04-08
Karen Richardson pose la question :
I need a formula for area that works for a square, a rectangle, a parallelogram, a trapezoid, and a triangle.
Jack LeSage, Chris Fisher and Harley Weston lui répond.
Area of an irregular shaped objects. 1999-04-07
Jesse Townsend pose la question :
How do I find the surface area of an irregulary shaped object such as someone's knee from the thigh to shin?

Thanks.
Walter Whiteley lui répond.

Resources for real-world math activities 1999-03-26
Kate O'Brien pose la question :
Where is there a collection of math acitivities or projects to use in high school Algebra I, Algebra II, or Trigonometry that tie concepts to real-world careers?
Jack LeSage lui répond.
John Napier and e 1999-02-06
Shimin pose la question :
I would like to ask about the exact definition of e, its history (like how John Napier came about discovering it) and its applications in problems and real life situations. Thank you!
Chris Fisher lui répond.
Graphing the Derivative 1999-01-18
Milena Ghebre pose la question :
This question has been nagging me for sometime now.

Is there a way of finding out the derivative of a function, just by looking at the graph of it?
Walter Whiteley lui répond.

Complex numbers and the quadratic formula 1998-12-25
Richard Peter pose la question :
My age is 16, and my name is Richard. My question relates to the topic complex numbers & the quadratic formula.

I would like to know how to solve quadratic equations in which the discriminant is less than 0 (i.e. we get two complex solutions to the quadratic)

3x2+2x+5 = 0

and how mathematicians like euler contributed to this field. If it would be possible I would also like to know how this type of quadratic equations can be graphed?
Harley Weston lui répond.

The bricklayers formula 1998-11-24
Rachel Kaplan pose la question :
I have to do a report on the bricklayers formula N = 7LH. Can you give me any information on this.
Harley Weston lui répond.
Intersection of planes 1998-11-22
Dave Rasmussen pose la question :
I am a teacher of secondary mathematics with a question about the uses of Three Dimensional Co-ordinate Geometry. I have been teaching my students to write equations of planes and lines, - to find the intersection of these and the distance between them. What I am having difficulty finding are good applications of these techniques to "real world" situations. Can anybody help me?
Walter Whiteley and Harley Weston lui répond.
Logs 1998-10-21
Benny pose la question :
What does log means? How come we have to use log
Chris Fisher lui répond.
A Kite 1998-10-07
Paul Scott pose la question :
What is the mathematical term for the kite shape?
Walter Whiteley lui répond.
Points on a Graph 1998-10-03
Nouver Cheung pose la question :
If the point P(-3,2) is on the graph of y=f(2x-1), what point must be on y=f(x)-3?
Harley Weston lui répond.
Radicals 1998-09-15
Lana Sabo pose la question :
Question:
fifteen times the square root of twenty, divided by the square root of 2.

nine subtract the square root of forty-five, divided by 3.

the square root of 18 plus the square root of 12, divided by the square root of 3.
Harley Weston lui répond.

Snake Eyes 1998-07-20
Would you please advise me what the probability is of rolling snake eyes on a pair of dice is? My recollection is 1 out of 6 multiplied by 1 out of 6 = 1 out of 36 is this correct?
Jack LeSage and Penny Nom lui répond.
Graph question 1998-05-12
Rose Seminary pose la question :
Why is the point of intersection of two lines the solution to the corresponding system of equations?
Penny Nom lui répond.
Isosceles trapezoid formula 1998-05-12
Donna McMullin pose la question :
The teacher of Gifted and Talented Math has been trying to locate the formula for anisosceles trapezoid and we can't find it anywhere. Could it be the same formula for that of a parallelogram ? Please advise.
Walter Whiteley lui répond.
Graph Distortion 1998-02-19
Dana Steffan pose la question :
What exactly is graph distortion? I have to explain it for a project and I can't seem to find anything on it.
Penny Nom lui répond.
Two Inscribed Trapezoids 1998-01-27
James pose la question :
A hexagon inscribed in a circle has three consecutive sides each of length 3 and three consecutive sides each of length 5. The chord of the circle that divides the hexagon into two trapezoids, one with three sides each of length 3 and the other with three sides each of length 5, has length equal to m/n, where m and n are relatively prime positive integers. Find m+n.
Haragauri Gupta lui répond.
The two-digit numbers from 10-99 1998-01-21
Alexis Riddle pose la question :
My name is Alexis. I'm a student in 8th grade and I'm taking Algebra I Honors. My question is: In a string of numbers, two adjacent digits are considered as a two-digit number. For instance, the string 11012 contains the numbers 10, 11, and 12. What is the number of digits in the smallest string that contains all of the two-digit numbers from 10-99? Please help and thank you for any assistance you can give me.
Penny Nom lui répond.
Quadratic Graphs 1998-01-13
Simon Batten pose la question :
How does varying the values a,b and c affect the graph of
y=ax2 + bx + c?

Harley Weston lui répond.
(-2)^x 1997-12-12
Ken Bokinac pose la question :
During our math class last week we came accross an equation that we could not graph we were wondering if you could help us try and graph it. The question is:

y=(-2)^x
Harley Weston lui répond.

The Area of a Trapezoid. 1997-05-07
Mary George pose la question :
I am doing the area of trapezoids and mixed polygons and I was wondering if you can help me figure out this problem.

A line segment drawn parallel to a leg of a right triangle divides the other leg into segments of 3cm and 6 cm and the hypotenuse into segments of 5cm and 10 cm. The two figures formed are a triangle and a trapezoid. Find the area of each.

I would appreciate if you would email me back the solution.
Harley Weston lui répond.

Graphing Inequalities of Conic Sections 1997-03-24
James Sheldon pose la question :
I'm trying to graph Systems of Conic Sections with inequalities, but I'm running into problems on which area to shade:
x^2+y^2 is greater than or equal to 16
xy > 4

So I graph these two equations, and then my teacher said to substitute a point into it but I'm still not sure how to do it...
Penny Nom lui répond.

Approximating pi. 1996-11-04
Ben Dixon pose la question :
How do you calculate Pi? Do you have to somehow combine the equation for a circle with the formula for the circumference?
Chris Fisher lui répond.
Approximating roots. 1996-11-04
Ben Dixon pose la question :
How do you calculate a square root? eg the square root of 2.

There is obviously some sort of successive approximation type algorithm for doing it to however many decimal places is required, but what is the algorithm?
Harley Weston lui répond.

Where do 2^x and x^2 intersect? 1996-07-23
Darin M Selfridge pose la question :
The intersecting points of y=2^x and y=x^2, graphed along with x^2=2^x, are obviously three... (2,4), (4,16) and (?,?). I am having problems figuring out the value, and even an equation for the (?,?). Can you help?
Harley Weston Chris Fisher lui répond.
Show that this construction yields a rhombus. 1996-06-16
LennyB pose la question :
Hello, I have a problem. I doubt you can help me. If you have an isosceles trapezoid, and you connect the midpoints of the four sides of the isosceles trapezoid forming a quadrilateral, how do you prove that it's a rhombus in a 2 column proof??
Penny Nom lui répond.
How do you graph these inequalities? 1996-05-17
Robert pose la question :
How do you graph the inequalities 5+gg, and g+h>6 on one coordinate plane?
Harley Weston and Maxine Stinka lui répond.
How do you graph these equations? 1996-04-30
David Gamble pose la question :
How do you graph the equation x=3 on a x,y graph? Also how do you graph f(x)=-3x + 2 on the same graph?
Harley Weston and Maxine Stinka lui répond.
An application of Pythagoras' theorem 1996-04-09
Mike pose la question :
We'd like to know what practical applications there may be for the Pythagorean theorem.
Penny Nom and Maxine Stinka lui répond.
 
Page
1/1

 

 


Centrale des maths reçoit une aide financière de l’Université de Regina et de The Pacific Institute for the Mathematical Sciences.

CMS
.

 

accueil centre de ressources accueil Société mathématique du Canada l'Université de Regina PIMS